You are on page 1of 40

CHÀO MỪNG NGÀY 20/11

Tháng 11, tiết trời đã trở lạnh sang đông, khiến ta quên đi ánh nắng mặt trời gay gắt, những cành
phượng vĩ đỏ thắm bầu trời những lúc vào hạ, cũng không còn nữa thoáng khẽ khàng mang theo chút
man mác se lạnh cuối thu… Nhưng giữa những ngày gió lạnh đầu mùa, trong mỗi chúng ta lại ấm lên cảm
giác bồi hồi, xúc động khó tả, nhớ về thầy cô giáo của mình, bởi ngày 20-11 lại đến!
20-11, Ngày Nhà giáo Việt Nam, ngày lễ của toàn ngành Giáo Dục, cũng là dịp để tôn vinh những người
thầy, người cô đã và đang đứng trên bục giảng truyền đạt tri thức và đạo làm người cho bao lớp học trò,
tôn vinh truyền thống hiếu học và truyền thống tôn sư trọng đạo của dân tộc ta: “Không thầy đố mày làm
nên” hay “Nhất tự vi sư, bán tự vi sư”…Chính thầy cô là những người đã chắp cánh cho mọi ước mơ của
chúng ta bay cao, bay xa, cung cấp hành trang kiến thức để chúng ta bước vào đời và thành công trong
cuộc sống…
Con đò mộc, mái đầu sương
Theo con đi khắp muôn phương mai này
Khúc sông ấy vẫn ngày ngày
Thầy đưa những chuyến đò đầy qua sông.
Thầy cô ơi! Công ơn lớn lao của thầy cô, chúng con không biết phải đền đáp như thế nào. Chúng con
biết, nhiều lúc đã làm thầy cô buồn lòng, không vui Chúng con xin hứa sẽ cố gắng học tập hơn nữa để có
thể phần nào bù đắp những lỗi lầm của mình. Cũng vẫn biết rằng, thầy cô đã phải vất vả như thế nào khi
lái một con đò, với biết bao nhiêu sóng gió, thầy cô vẫn một lòng vì chúng con, chở che và dìu dắt chúng
con. Những lúc chúng con chùn bước, bàn tay ấm áp của thầy cô lại nâng đỡ nhẹ nhàng. Những lúc
dường như bất lực, giọng nói truyền cảm ấy lại đến với chúng con, cho chúng con thêm nhiều nghị lực để
tiếp tục phấn đấu. Thầy ơi, cô ơi, ngàn lần chúng con xin cảm ơn người! Chúng con sẽ cố gắng lắng nghe
tiếng thời gian, nắm thật chặt trong tay dòng thời gian của mình, để có thể bước đến bến bờ thành công
như niềm hi vong mà thầy cô dành cho chúng con. Đất nước cho chúng con một quê hương để thương, để
nhớ. Cha mẹ cho chúng con một hình hài, dáng dấp để sống và học tâp. Và thầy cô cho chúng con một
hành trang vững chắc để bước vào đời.
Nhân ngày 20-11, ngày Hiến Chương Nhà Giáo Việt Nam, chúng con xin gửi đến thầy cô lời cảm ơn
chân thành với tấm lòng tri ân sâu sắc nhất, cùng những lời chúc tốt đẹp. Chúc thầy cô luôn khỏe và mãi
là những người đưa đò vĩ đại trên dòng sông tri thức của cuộc đời.
______________________________*______________________________
TẬP SAN TOÁN HỌC & SINH VIÊN Số 33, tháng 11 năm 2010
Chịu trách nhiệm xuất bản MỤC LỤC
SỞ TT&TT NGHỆ AN
 Tạo thói quen khai thác giả thiết…………….......5
Chịu trách nhiệm nội dung
 Toán học với sự phát triển kinh tế vĩ mô……….12
PGS.TS Nguyễn Thành Quang
 ĐH Vinh - nơi tạo dựng tương laic ho tuổi trẻ…18
 Kinh nghiệm học môn Tô-pô……………….......22
 Những định lý giải tích quan trọng……………..16
 Định lượng toán tử tuyến tính…………………..20
Trưởng ban: Trần Quốc Luật  Đề ra kỳ này…………………………………….22
Phó ban: Nguyễn Thúy Hằng  Tư duy hình học trong đại số…………………...25
BTV: Đinh Bích Yến, Nguyễn Duy Diện, Lê Như Hảo,  Khảo sát tính liên tục đều……………………....28
Hoàng Thị Ngọc Trà, Mai Thị Phương, Nguyễn Huy  Hướng dẫn sử dụng LaTeX………………...…..30
Hùng, Nguyễn Anh Sơn, Võ Viết Chương, Phan Hồng In 250 bản tại Xưởng in Đại học Vinh, khuôn khổ
19.27 cm. Giấy phép xuất bản số 171/2010/GPXB –
Quân, Nguyễn Thị Ngọc Hà, Bùi Văn Hoàng, Phạm STTTT cấp ngày 09/11/2010. In xong và nạp lưu
Thị Liên, Nguyễn Thanh Huyền, Phan Đình Hùng. chiểu tháng 11 năm 2010.
1

KẾT HỢP ĐÀO TẠO VÀ NGHIÊN CỨU KHOA HỌC


TẠI KHOA TOÁN - TRƯỜNG ĐẠI HỌC VINH
PGS.TS. NGUYỄN THÀNH QUANG
Thành công nổi bật nhất trong quá trình phát Trong những năm gần đây, có nhiều cán bộ
triển của Khoa Toán - Trường Đại học (ĐH) Vinh giảng dạy Khoa Toán đã báo cáo khoa học tại nhiều
là những thành tựu về công tác đào tạo và nghiên nước trên thế giới như: Mỹ, Pháp, Italy, Trung
cứu khoa học. Một số tập thể nghiên cứu trong Quốc, Thái Lan. Khoa đã thực hiện 12 đề tài nghiên
Khoa đã đạt được những kết quả mạnh, tập trung cứu cơ bản cấp Nhà nước, 25 đề tài cấp Bộ. Nhiều
vào một số hướng quan trọng, có ý nghĩa khoa học, giáo trình và sách chuyên khảo đã được xuất bản,
được nhiều nhà khoa học trong và ngoài nước đánh phục vụ cho công tác đào tạo sau đại học ngành
giá cao. Hiện nay, đội ngũ giảng viên toán của Toán trong và ngoài Trường ĐH Vinh.
Trường ĐH Vinh gồm: 1 giáo sư, 10 phó giáo sư, Tập thể sinh viên của Khoa đã đạt được thành
15 tiến sĩ. Khoa đang đào tạo 3 ngành đại học: Sư tích: 20 giải thưởng trong Hội thi sinh viên nghiên
phạm Toán học, Toán học, Toán - Tin học và ứng cứu khoa học của Bộ Giáo dục và Đào tạo, trong đó
dụng trên tổng số 600 sinh viên. Về đào tạo sau đại có 2 giải Nhất, 3 giải Nhì; 115 giải trong các kỳ thi
học, Khoa có 5 chuyên ngành đào tạo thạc sĩ và tiến Olimpic Toán học sinh viên toàn quốc (2000-2010)
sĩ: Toán Giải tích, Đại số và Lý thuyết số, Lý luận do Hội Toán học cùng Bộ Giáo dục và Đào tạo
và phương pháp dạy học bộ môn Toán, Hình học và đồng tổ chức, trong đó có 18 giải nhất. Sinh viên
Tôpô, Lý thuyết Xác suất và thống kê Toán học. Đã Nguyễn Trần Thuận khóa 46A đạt 2 giải nhất trong
có 40 nghiên cứu sinh của Khoa bảo vệ thành công kỳ thi năm 2009, trong đó có môn thi Đại số đạt
luận án tiến sĩ tại cơ sở đào tạo sau đại học - điểm tuyệt đối. Sinh viên Trần Quốc Luật khoá 50A
Trường Đại học Vinh, trong đó nhiều luận án bảo đạt Giải nhất môn Giải tích trong kỳ thi năm 2010.
vệ đạt loại xuất sắc; nhiều luận án trực tiếp do cán Hoạt động thi Olimpic toán hàng năm của Hội Toán
bộ trong Khoa làm hướng dẫn chính. học đã góp phần động viên lòng say mê toán học
Toàn Khoa đã có 171 công trình toán học công của các em sinh viên, góp phần tôn vinh trí tuệ toán
bố trên các tạp chí Toán học chuyên ngành có uy tín học của tuổi trẻ Việt Nam.
trong và ngoài nước. Nhiều giảng viên của Khoa có Bốn lớp bồi dưỡng cử nhân tài năng toán tại
công trình được liệt kê trong Tạp chí Mathematical Khoa đã được mở là một hình thức đào tạo chất
Reviews của Hội Toán học Mỹ, trong đó có 4 lượng cao, tạo địa chỉ tin cậy để tạo nguồn cán bộ
giảng viên có từ 10 công trình trở lên; 11 giảng viên cho Khoa. Lớp cử nhân tài năng khóa V đang triển
và nghiên cứu sinh có công trình công bố thuộc khai học 2 chuyên đề.
danh mục ISI. Về công tác quản lý đào tạo, Khoa Toán đã chỉ
Hiện tại, Khoa có 13 giảng viên với độ tuổi dưới đạo việc xây dựng nề nếp quản lý đào tạo qua các
35 đang theo học chương trình đào tạo tiến sĩ, trong chuyên ngành và tổ bộ môn, xây dựng chương trình
đó có 6 người đang làm nghiên cứu ở nước ngoài. chuẩn, tổ chức thực hiện chương trình đào tạo, viết

TOÁN HỌC & SINH VIÊN số 33


2
sách và giáo trình, rèn luyện phương pháp nghiên Khoa Toán thường xuyên tổ chức các hội nghị
cứu cho sinh viên và học viên, xây dựng thư viện, khoa học, nhằm giúp cho sinh viên, học viên được
mạng Internet tạo điều kiện cho sinh viên và học tiếp cận với hướng nghiên cứu mới, giao lưu với
viên tự học. các nhà khoa học đầu ngành, có cơ hội trình bày các
Song song với những nỗ lực kể trên, nhiều sinh báo cáo và định hướng nghiên cứu. Nhiều cán bộ
viên giỏi của Khoa bằng nhiều con đường khác của Khoa đã có mối quan hệ hợp tác nghiên cứu
nhau đã được gửi đi đào tạo thạc sĩ ở các trường đại khoa học thường xuyên với các nhà toán học trong
học nước ngoài. Các trường đại học của Mỹ, Nga, và ngoài nước.
Trung Quốc, Đức, Pháp đã tiếp nhận NCS của Cùng với nhiều thành tích đã đạt được, hoạt
Khoa. Chương trình hợp tác Hỗ trợ đào tạo các nhà động đào tạo ngành Toán vẫn còn nhiều hạn chế
toán học trẻ Việt nam (Formathvietnam) đã cấp cho cần khắc phục:
Khoa 6 học bổng sau tiến sĩ tại Pháp. Có 01 NCS đã - Trình độ đầu vào của sinh viên, học viên không
bảo vệ thành công luận án dưới sự đồng hướng dẫn đồng đều. Sinh viên những năm đầu chưa tham gia
của các nhà toán học hai nước Pháp - Việt. Tổ chức nhiều hoạt động nghiên cứu, vẫn còn nhiều sinh
Formathvietnam cũng đã tổ chức tại Khoa Toán - viên chưa thực sự say mê trong học tập. Các ảnh
Trường Đại học Vinh các trường toán ngắn hạn. hưởng mặt trái của cơ chế thị trường vẫn còn ít
Các giáo sư và các nhà toán học tên tuổi như nhiều tác động đến người học, đã làm cho nhiều
Cachier, Mutsuo Oka, Feréderic Phạm, Nguyễn người thiếu nhiệt tình khám phá trong các nghiên
Thanh Vân, Lê Dũng Tráng, ... đã tới đọc bài giảng cứu cơ bản có ý nghĩa khoa học lâu dài. Việc cập
khoa học cho cán bộ, NCS và học viên cao học tại nhật và trao đổi các kiến thức tin học trong sinh
Khoa. Năm 2010, Tổ chức Rencontres du Vietnam viên còn yếu; học tập ngoại ngữ chưa thường xuyên
(Gặp gỡ Việt Nam) do Giáo sư Trần Thanh Vân liên tục, kém hiệu quả; khả năng tự học chưa cao.
làm chủ tịch, đã trao 7 học bổng Odon Vallet cho 2 Sinh viên khai thác tài liệu thư viện (đặc biệt tài
NCS và 5 sinh viên của Khoa có thành tích xuất sắc liệu tiếng nước ngoài), sử dụng hiệu quả công cụ
trong học tập, với tổng trị giá 53 triệu đồng. Từ năm mạng Internet, các phần mềm tin học trong học tập,
2008, Khoa đã ký kết một hợp tác đào tạo và nghiên nghiên cứu còn ít.
cứu với Trung tâm Vật lý Lý thuyết Quốc tế Triese, - Đội ngũ cán bộ hướng dẫn chính luận án tiến sĩ
Italy (ICTP) và trong khuôn khổ hợp tác này đã có của Khoa còn mỏng, trong Khoa chưa có nhiều mũi
3 giảng viên trẻ của Khoa Toán được đi thực tập nhọn nghiên cứu cơ bản đủ mạnh để có thể hội nhập
khoa học tại ICTP. khu vực và quốc tế về đào tạo tiến sĩ.
Cơ sở đào tạo sau đại học trường ĐH Vinh đã - Số giáo trình đã biên soạn và nghiệm thu đưa
mời được hơn 100 nhà toán học từ Viện Toán học, vào sử dụng còn ít, chưa đáp ứng đầy đủ nhu cầu
ĐHQG Hà Nội, ĐHSP Hà Nội, Đại học Huế, Viện học tập của sinh viên, học viên.
Khoa học Giáo dục,... tham gia giảng dạy, hướng - Thông tin phản hồi hai chiều giữa người dạy và
dẫn luận văn thạc sĩ, luận án tiến sĩ. người học chưa được chú trọng. Vẫn còn nặng về

TOÁN HỌC & SINH VIÊN số 33


3
truyền thụ kiến thức cụ thể, không làm nổi bật được 4. Tạo ra một môi trường làm việc chính quy cho
kiến thức gốc; nhiều người học không nắm được mảng đào tạo: Trang bị phòng làm việc cho các bộ
bản chất của các khái niệm cơ bản của toán học dẫn môn; thư viện, Internet công cộng miễn phí; phòng
tới không ứng dụng được toán học vào các ngành bảo vệ luận văn, luận án,… để cán bộ, sinh viên,
khoa học khác và thực tiễn đời sống. học viên có điều kiện học tập và nghiên cứu tốt
- Nhiều sinh viên, học viên còn tâm lý ỷ lại, lười hơn.
đào sâu suy nghĩ trong học tập và nghiên cứu; nặng 5. Cần có chính sách cụ thể về hỗ trợ kinh phí
học để mà thi chứ không học để mà biết, vận dụng. cho sinh viên, học viên có kết quả nghiên cứu, để
Từ thực tế đào tạo ngành toán tại Khoa trong thời họ có thể tham gia các đề tài, hội nghị, hội thảo
gian qua, Khoa có những đề xuất sau: khoa học trong và ngoài nước; khen thưởng đối với
1. Chất lượng đào tạo và nghiên cứu của đội ngũ những sinh viên, học viên có thành tích nghiên cứu
các thầy giáo là tiền đề cần thiết và là niềm cảm tốt và cán bộ hướng dẫn thành công khoá luận, luận
hứng về tư duy sáng tạo cho các đồng nghiệp trẻ và văn, luận án có kết quả xuất sắc.
sinh viên và chính điều này đảm bảo tính bền vững 6. Tinh giản nội dung giảng dạy theo xu thế hiện
cho uy tín của Trường. Do đó, phải tăng cường vai đại, hội nhập; tăng cường thời gian tự học và tự
trò quản lý, giám sát của bộ môn, khoa chuyên nghiên cứu của người học. Điều đó đặt ra một
ngành đối với đào tạo và sinh hoạt khoa học của nhiệm vụ cho chúng ta là công tác đào tạo cần phải
giảng viên, hoạt động chuyên môn của học viên ở có một sự đổi mới mạnh mẽ về nội dung chương
những nội dung rất cụ thể: Kế hoạch làm việc, sinh trình, phương pháp giảng dạy và học tập, biên soạn
hoạt chuyên môn, nội dung và chương trình giảng giáo trình cũng như công tác quản lý đào tạo. Cần
dạy. Thực hiện nghiêm túc các khâu: xét duyệt đề có cơ chế chính sách thu hút giáo sư giỏi từ các
tài; bảo vệ đề cương nghiên cứu của NCS; đánh giá trường đại học, viện nghiên cứu trong và ngoài
luận văn, luận án. Trong luận văn thạc sĩ và luận án nước tham gia giảng dạy, hướng dẫn, đồng hướng
tiến sĩ, với những cấp độ khác nhau cần xác lập cân dẫn và đánh giá khóa luận, luận văn, luận án cho
đối giữa 3 nội dung: Lý thuyết - Nghiên cứu - Ứng học viên và sinh viên.
dụng. 7. Huy động toàn bộ hệ thống tổ chức và chính trị
2. Đẩy mạnh ứng dụng công nghệ thông tin vào trong Khoa giúp đỡ sinh viên học tập theo hệ thống
việc đổi mới phương pháp giảng dạy, đặc biệt tín chỉ: Tư vấn học tập, đăng ký học phần, tài liệu
khuyến khích cán bộ và sinh viên, học viên sử dụng học tập, phương pháp học tập, hướng dẫn ôn thi học
các phần mềm tin học trong học tập, giảng dạy, phần,...
nghiên cứu toán học. Trong thời gian tới, Khoa Toán sẽ cố gắng có
3. Cần có kế hoạch cụ thể về đào tạo đội ngũ cán những bước đột phá mạnh hơn, nhanh hơn để tạo ra
bộ trẻ, giỏi, có tâm huyết để đảm đương khối lượng một số mũi nhọn trong đào tạo và nghiên cứu khoa
lớn các chuyên đề cơ bản ở các chuyên ngành, đón học, hướng tới có thể hội nhập được với các trường
đầu hợp tác quốc tế về đào tạo và nghiên cứu. đại học trong khu vực và quốc tế.

TOÁN HỌC & SINH VIÊN số 33


4

TẠO THÓI QUEN KHAI THÁC GIẢ THIẾT BÀI TOÁN


TRẦN QUỐC LUẬT – 50A Toán

Trong đề thi Đại học Khối A, năm 2010 có bài toán


“phân loại” như sau:
 16 x 4  25 x 2  5  8  
3  4x 1  0
BÀI TOÁN: Giải hệ phương trình: 16 1  2 x 
 
 4 x2  1 4x2  5   0
* :  2 2
 
 4 x 2  1 x  ( y  3) 5  2 y  0 1 3  4x 1
 2  16 
 4 x  y  2 3  4 x  7.


  2 x  1  2 x  1 4 x 2  5 
3  4x 1

0
3 5
Trước hết, ta đặt điều kiện x  , y  . Rõ ràng 1 3
4 2  x  (do 0  x  ).
đây là một hệ phương trình không mẫu mực. Ta hãy 2 4
xem xét từng phương trình của hệ. Nhận thấy 3
Cách 2.2: Nhận thấy x  0 và x  đều không phải
phương trình 1 có 2 ẩn “phân ly” “rời nhau” (có 4
thể “cô lập” mỗi ẩn sang một vế của phương trình), 3 5  4 x2
là nghiệm nên 0  x  . Thay y  vào
đồng thời chứa một biểu thức trong dấu căn (hơn nữa 4 2
bậc của x và y bằng nhau và bằng 3). Ta khai phương trình thứ hai ta được
2
thác triệt để những điều này như sau. Trước hết “cô
4x2 
5  4x  2

 2 3  4 x  7.
lập” mỗi ẩn về mỗi vế ta được
4
 4x 2

 1 x  (3  y ) 5  2 y . Đặt t  5  2 y , ta có  3
Trong khoảng  0;  hàm số

t t 1 2
  4

x 4x2  1   2
hay    
2x 4 x2  1  t t 2  1 .
5  4x  2
2

Đến đây ta đã phát hiện ra hàm đặc trưng g ( x)  4 x 2   2 3  4 x nghịch biến.


4
f (t )  t (t 2  1), rõ ràng f '  x   3t 2  1  0, t  R Thật vậy, ta có
nên hàm này đồng biến nên ta có t  2 x hay 4
g ( x )  4 x(4 x 2  3)   4 x(4 x 2  3)  0.
2x  5  2 y .  3 3  4x
Với bản chất như vậy, ngoài cách trình bày trên ta 1
Mặt khác ta lại có g    7. Vì vậy phương trình
còn có thể trình bày bước này như sau: 2
Cách 1.1: Đặt 5  2 y  2t  0, ta có ngay 1 trở 1
g ( x )  7 chỉ có một nghiệm duy nhất là x  .
thành x(4 x 2  1)  t (4t 2  1). 2
Cách 2.3: Viết lại  4  dưới dạng f ( x )  g ( x) với
 
Suy ra  x  t   4 x 2  xt  t 2  1  0  x  t.
 
f ( x)  8 3  4 x và g ( x )  16 x 4  25 x 2  3. Khảo
Cách 1.2: Ta có:
3  3
 2  5y 
3 2  5y sát riêng lẻ f ( x), g ( x ) thấy trên khoảng  0;  hàm
1  4 x  x  4   
  4
 2  2
f ( x ) nghịch biến và g ( x) đồng biến nên  4  có
 2x  5  2 y. nghiệm duy nhất.
Vấn đề đã được giải quyết một nửa, ta sẽ xử lý Cách 2.4: Đặt g ( x)  4 x 2  2 3  4 x .
phương trình  2  với điều kiện  3 . 4
Ta có g ( x )  8 x   0 với mọi x  0.
Cách 2.1: Thay  3 vào  2  ta có: 3  4x
2 1
 5  4x2 
2 +) Nếu x  thì từ  3 ta có y  2. Do vậy
4x     2 3  4x  7 2
 2  1
 16 x 4  25 x 2  8 3  4 x  3  0  4 g ( x )  y 2  g ( )  4  7 , hệ * vô nghiệm.
2

TOÁN HỌC & SINH VIÊN số 33


5
1
+) x  thì y  2 , thỏa mãn.
2 hay 16 x 4  24 x 2  8 3  4 x  3  0.
1 Đặt 3  4x  t thì t [0; 3] .
+) Nếu x  thì từ  3 ta có y  2. Do vậy
2 Khử x ta có: t 8  12t 6  30t 4  36t 2  128t  183  0
1
g ( x )  y 2  g ( )  4  7 , hệ * vô nghiệm.
2  
  t  1 t  3 t 6  2t 5  5t 4  4t 3  7t 2  2t  61  0
2 2
Cách 2.5: Thay 2 x  5  2 y vào  2  ta được  
 t  1 (vì t4  t 1  6 9  t4  4t3  6t2   t 1  6  0

( y  1) 2  2 3  2 5  2 y  3. với mọi t [0; 3] ).


Bài tập tương tự áp dụng:
Đặt a  y  1 thì ta được a 2  2 3  2 3  2a  3 Giải các hệ phương trình sau:
hay a  3  2 3  2 3  2a . 2 x 2  6 x  5  4 y  3
1. 
2 4 3 2
Đặt b  3  2a ; c  3  2b ta có a  3  2c .  x  y  2 y  5 y  4 y  4.

( x 1) x  2x  5  4 y y  1  2(2 y  x  1)


2 2
Cách trình bày 1: 2. 
+) Nếu a  1 thì b  1, nên c  1, do vậy a  1 , vô  x3  2 y  3  2 x  2.
lý.
+) Nếu a  1 thì b  c  1 , thỏa mãn. Thay vào ta có *********************************************

1
nghiệm của hệ * là x  ; y  2.
CON VỚI THẦY
2 Phan Hồng Quân (sưu tầm)
+) Nếu a  1 thì b  1, nên c  1, do vậy a  1, vô lý.
Con với thầy
Cách trình bày 2:
Người dưng nước lã
Không mất tính tổng quát, ta giả sử a  b  c.
Con với thầy
Từ a  b có a  3  2a , nên a  1. Tương tự b  1. Khác nhau thế hệ…
Do vậy b  1  3  2a . Đẳng thức xảy ra khi
a  b  1. Từ đó a  b  c  1. Đã nhiều lần tôi tự hỏi mình
3 Mười mấy ngàn ngày không gặp lại
Cách 2.6: “Làm chặt” điều kiện được 0  x  ; Những thầy giáo dạy tôi ngày thơ dại
4
5 Vẫn bên tôi dằng dặc hành trình…
0  y  . Chú ý với điều kiện này thì các hàm
2
Vẫn theo tôi những lời động viên
 
f1 ( x )  x 4 x 2  1 ; g 2 ( y )  y 2 đồng biến, đồng thời
Mỗi khi tôi lầm lỡ
f 2 ( y )   3  y  5  2 y và g1 ( x )  4 x 2  2 3  4 x Vẫn theo tôi những lời nhắc nhở
nghịch biến. Ta đánh giá: Mỗi khi tôi tìm được vinh quang...
1
Với x  thì từ 1 ta có f 2 ( y )  1  y  2. Từ
2 Qua buồn vui, qua những thăng trầm
 2   g1 ( x)  3  g 2 ( y)  4  y  2 (mâu thuẫn). Câu trả lời sáng lên lấp lánh
1 1 Với tôi thầy ký thác
Với x  tương tự ta có mâu thuẫn. Vậy x  , Thầy gửi tôi khát vọng người cha…
2 2
thay vào thấy thỏa mãn.
Đường vẫn dài và xa
Cách 2.7: Khử y ta có: Thầy giáo cũ đón tôi từng bước!
2
 5  Từng bước một, tôi bước
4 x2  2 3  4 x   2  x2   7
 2  Với kỷ niệm thầy tôi...

TOÁN HỌC & SINH VIÊN số 33


6

SUY NGHĨ VỀ MỘT BÀI TOÁN


HAI KHÔNG GIAN TÔPÔ ĐỒNG PHÔI
BÙI XUÂN QUANG – ĐH Hải Phòng

Tóm tắt. Bài viết phân tích hướng chứng minh và kết quả mà không chứng minh chi tiết, phần còn lại
một vài suy nghĩ sau khi giải một bài toán về hai xin dành cho bạn đọc.
không gian Tôpô đồng phôi. Tôi giải bài toán này như sau: Thay vì xét hình cầu
Mùa hè vừa qua tôi có may mắn được tham dự S 2 , xét mặt cầu:
Trường hè Toán học cho Sinh viên tổ chức tại Viện
Toán học. Tôi đăng kí lớp Tôpô (Giảng viên là PGS. S  x, y, z   R 3

: x2  y2  z2  z .
TS. Hà Huy Vui). Bài tập sau đây là một trong những và chứng minh rằng S\{N} (với N là một điểm nào
bài mà Thầy cho chúng tôi về nhà: đó nằm trên S ) đồng phôi với R 2 . Khi đó trong lớp
Chứng minh rằng mặt cầu đơn vị S 2 ( S 2 là mặt Tôpô chúng tôi có một bạn thắc mắc là đề bài Thầy
cầu đơn vị trong R 3 ) nếu bỏ đi một điểm thì đồng cho là S 2 , tại sao lại chứng minh cho S thì Thầy
phôi với R 2 . trả lời là: “Về mặt tôpô thì hai mặt cầu đó giống
Bài toán này không phải là quá phức tạp, tuy nhau”. Chúng ta hãy cùng lý giải sự giống nhau đó.
nhiên nó lại chứa đựng nhiều điều thú vị. Bài viết Cũng cần nói luôn là do mục đích của bài báo nên có
này tôi sẽ trình bày một số suy nghĩ của mình về bài thể ban đầu bạn đọc thấy sự trình bày không được tự
toán trên. Trước hết, tôi xin được nêu lên không nhiên lắm, sự không tự nhiên đó sẽ được lý giải ở
chứng minh vài khái niệm và Định lý có liên quan cuối bài viết.
trong việc trình bày để những ai chưa học Tôpô dễ 1. Đầu tiên tôi dựng mặt cầu
theo dõi:  1
B=  x; y; z   R 3 : x 2  y 2  z 2   . Ta thấy B có
1. Giả sử X là một tập hợp khác rỗng,  là một họ  4
những tập con của X (   P( X ) ).  được gọi là một tâm là gốc O và nằm bên trong hình cầu S 2 . Ta xét
cấu trúc Tôpô trên X nếu nó thỏa mãn các điều kiện phép tương ứng sau đây (xem hình vẽ):
sau đây: Từ gốc toạ độ O, ta kẻ tia
i) Tập X và tập rỗng đều thuộc  . Ot bất kỳ cắt B tại M, cắt
ii) Giao hữu hạn các tập thuộc  thì
S 2 tại M ' . Khi đó, bạn
thuộc  . đọc có thể kiểm tra một
iii) Hợp bất kì các tập thuộc  thì thuộc cách chi tiết rằng,
. ánh xạ sau đây:
Tập hợp X cùng với một cấu trúc tôpô trên nó được
f : B  S2
gọi là một không gian Tôpô (Topological space).
2. Ánh xạ f : X  Y giữa hai không gian tôpô X và M  f M   M '
Y được gọi là liên tục tại x0  X nếu với mọi lân cận là một phép đồng phôi.
V của f  x0  đều tồn tại một lân cận U của x0 sao
2. Tiếp theo, ta tịnh tiến mặt cầu B theo trục Oz để
cho f U   V . 1

3. Giả sử f : X  Y là ánh xạ giữa hai không gian tâm của mặt cầu này trùng với điểm  0;0;  . Ta gọi
 2
tôpô X và Y. Khi đó, f được gọi là một phép đồng mặt cầu này là S, suy ra
phôi nếu f là song ánh và ánh xạ ngược f 1 của nó
cũng liên tục. Lúc đó ta cũng nói không gian tôpô X
S   x, y, z   R 3

: x2  y2  z2  z .

đồng phôi với không gian tôpô Y, và kí hiệu là Lúc này ta có ngay S  B (do “  ” là một quan hệ
tương đương như trong phần lý thuyết ta đã nói).
X  Y.
4. Quan hệ đồng phôi giữa các không gian tôpô là
3. Vấn đề còn lại là phải chứng minh cho S bỏ đi
quan hệ tương đương.
5. Phép đẳng cự là phép đồng phôi (nhưng ngược một điểm nào đấy thì đồng phôi với R 2 . Phép chứng
lại thì không luôn đúng). minh được thực hiện như sau (hình vẽ):
Bây giờ chúng ta trở lại bài toán. Cũng xin nói
luôn là do khuôn khổ bài báo nên tôi chỉ đưa ra các

TOÁN HỌC & SINH VIÊN số 33


7
Rõ ràng N  0;0;1  S. Với mỗi P(x;y;z)  S\{N} ta
kẻ tia Pj cắt R 2 tại Q(x;y). Với một sự kiên trì và cẩn
LẦN ĐẦU XA NHÀ
Nguyễn Thị Ngọc Hà (sưu tầm)
thận thích đáng, bạn đọc có thể kiểm tra được ánh xạ
sau: Lần đầu tiên con đi học xa nhà
 : S\{N}  R 2 Bỏ lại đằng sau quê hương vời vợi nhớ
Xe lăn bánh mẹ còn thao nhắc nhở
P    P  Q
Nước mắt con chạm má tự bao giờ
là một phép đồng phôi. Học xa nhà con mới biết làm thơ
Đến đây, ta coi như bài toán đã được giải quyết. Nỗi nhớ mẹ tràn trên trang giấy trắng
Nếu suy nghĩ thêm một chút nữa về bài toán, tôi tin Con ở một mình trong căn phòng vắng lặng
sẽ có điều gì đó thú vị, bây giờ ta hãy đồng nhất R 2 Tiếng mẹ lại vọng về cố gắng nhé con
với không gian phức C (như cách đã biết trong Giải Học xa nhà con lại sợ những hoàng hôn
tích phức). Ta thấy khi P dần tới N thì tia Pj dần tới Cứ thấy nhà ai quây quần ăn cơm tối
vị trí song song với C . Vậy ta có thể coi N  S Con giật mình nước mắt rơi vội
tương ứng với điểm xa vô tận  của mặt phẳng phức, Mặn chát bờ môi, nghẹn đắng tâm hồn
mặt phẳng phức có bổ sung thêm điểm xa vô tận Học xa nhà con thấy mình lớn khôn
được gọi là mặt phẳng phức mở rộng và chúng ta kí Là lần đầu xa vòng tay của mẹ
hiệu là C . Như vậy, tương ứng  như trên được mở Dù ở đâu con cũng thấy mình nhỏ bé
Mỗi khi trở về bên mẹ yêu thương!
rộng tới đồng phôi giữa S và C (hay R 2 cũng vậy)
*********************
bằng cách đặt   N    . Đến đây, tôi tin là chúng ta
có thể hiểu thêm được bản chất của điểm vô cùng
LỜI RU CỦA THẦY
Phan Hồng Quân (sưu tầm)
trên không gian phức. Cuối cùng, lúc nãy là ta thiết
lập“sự đồng phôi” của S\{N} với R 2 bằng hình học Mỗi nghề có một lời ru
Dở hay thầy cũng chọn ru khúc này
(đồng nhất R 2 với C ), để có thể tính toán bằng giải
Lời ru của gió màu mây
tích, chúng ta cần thay đổi thứ tự và kí hiệu một chút
Con sông của mẹ đường cày của cha
để lời tính toán được dễ dàng hơn: Trong không gian
Bắt đầu cái tuổi lên ba
Euclide R 3 với hệ toạ trực chuẩn  O; , ,   , xét mặt Thầy ru điệp khúc quê nhà cho em
cầu S:  2   2   2   . Ứng với mỗi điểm Yêu rồi cũng nhớ yêu thêm
Tình yêu chẳng có bậc thềm cuối đâu!
z   x; y   C ta nối nó với N  S. Đường thẳng này Thầy không ru đủ nghìn câu
 x y z
2
 Biết con chữ cũng đứng sau cuộc đời
cắt S tại điểm T   ; ; 
1 z 2
1 z
2
1 z
2
 Tuổi thơ em có một thời
  Ước mơ thì rộng như trời, ngàn năm
và ta cần chứng minh ánh xạ Như ru ánh lửa trong hồn
 : C  S \{N} Cái hoa trong lá, cái mầm trong cây
2
 x y z  Thầy ru hết cả mê say
 x; y    2
; 2
; 2

 Mong cho trọn ước mơ đầy của em.
1 z 1 z 1 z  Mẹ ru em ngủ tròn đêm
là đồng phôi. Ngược lại, khi Thầy ru khi mặt trời lên mỗi ngày
T  N ta thực hiện phép giới Trong em hạt chữ xếp dày
hạn khi   1 ta được Đừng quên mẹ vẫn lo gầy hạt cơm
z   x; y    . Vậy là Từ trong vòm mát ngôi trường
Xin lời ru được dẫn đường em đi
chúng ta đã chứng minh
(Con đường thầy ngỡ đôi khi
bằng giải tích sự đúng đắn
Tuổi thơ lăn một vòng bi tới rồi!)
của bài toán.
Hẳn là thầy cũng già thôi
Kết thúc bài báo, cho phép em được gửi tới Ban tổ Hóa thân vào mỗi cuộc đời các em
chức Trường hè Toán học cho Sinh viên 2010 lời Thì dù phấn trắng bảng đen
cảm ơn sâu sắc nhất vì đã tạo mọi điều kiện thuận lợi Hành trang ấy đủ thầy đem theo mình.
cho chúng em học tập và giao lưu trong suốt thời
gian ở Viện Toán học và ĐHSPHN.

TOÁN HỌC & SINH VIÊN số 33


8

BÍ QUYẾT NÓI CHUYỆN TRƯỚC ĐÁM ĐÔNG


BÙI VĂN HOÀNG – 50 Toán Tin học - Ứng dụng

Phần lớn mọi người xem việc phải nói chuyện nhóm cá nhân riêng lẻ. Hãy cố gắng nhìn vào một ai
trước đám đông là nỗi sợ hãi kinh khủng nhất. Nỗi sợ đó một lúc. Khi nói chuyện với khán giả, tiếp thu
đó còn hơn cả sợ rắn, sợ đi máy bay, hoặc ngay cả những ý kiến phản hồi của họ để hoàn thành bài nói
sợ cái chết. Nhưng chúng ta, không thể trốn tránh nó chuyện của mình. Bạn sẽ vượt qua thôi mà! Tôi chưa
mãi. Nhiều người trong chúng ta có thể được mời ra bao giờ nghe thấy có ai chết trên bục diễn thuyết cả.
trình bày một báo cáo, phát biểu trong buổi họp phụ Bạn cũng không bị thở dốc, hụt hơi, quên mất tên
huynh học sinh, nói lời chúc mừng trong lễ cưới. mình hay nổi nóng. Đấy là những chuyện gây ám ảnh
Làm sao để bạn có thể vượt qua những thử thách đó? cho bất cứ ai phải đứng trước đám đông. Người ta
Thật đơn giản chỉ với một chút thời gian luyện tập. gọi đó là cơn ác mộng của diễn viên. Việc đó hoàn
toàn bình thường. Sử dụng một số kỹ thuật thư giãn
Quy tắc quan trọng nhất trong việc nói chuyện trước khi bắt đầu. Bạn có thể tìm một nơi để nhảy lên
trước đám đông là bạn phải biết mình nói gì. Điều nhảy xuống hoặc dậm chân thật manh, điều này sẽ
này nghe có vẻ ngớ ngẩn, nhưng thông thường các giúp bạn cảm thấy vững vàng và giảm bớt căng
“diễn giả” không hề có một ý niệm rõ ràng về những thẳng. Lắc bàn tay và co duỗi nắm tay. Điều này sẽ
gì họ truyền đạt đến người nghe. Bạn cần phải biết làm tay bạn bớt run. Nếu run tay thực sự là một vấn
chính xác bạn sẽ đưa người nghe đến đâu. Một khi đã đề thì hãy nắm lấy một tấm danh thiếp hay nắm vào
biết, hãy liệt kê nó thành 3 hay 4 điểm chính và soạn bục diễn thuyết khi nói chyện. Lè lưỡi ra, trợn mắt và
bài nói của mình tập trung vào những điểm này thôi. há miệng to hết cỡ, sau đó nhăn tít mặt lại. Việc này
Bạn không phải là một cuốn từ điển sống, việc đưa ra sẽ làn thư giãn các cơ mặt của bạn. Hít thật sâu và
quá nhiều thông tin hay không đủ thông tin cũng đều thở mạnh ra tiếng để làm ấm giọng của bạn. Tưởng
dở như nhau. Thực hành, nhưng không cần quá tượng như bạn đang ở trên một đám mây, không gì
nhiều: Liệt kê ra những gì bạn sẽ nói và tập nói 1 hay có thể làm hại đến bạn khi bạn đang ở trong đó. Hãy
2 lần. Sẽ rất hay nếu như bạn canh thời gian trong khi cố gắng giữ hình ảnh ấy trong đầu khi bạn đang đứng
tập, việc đó sẽ giúp bạn kiểm soát được thời gian nói trên diễn đàn. Nói chuyện trước công chúng càng
mà không sợ bị lố. Có thể sẽ có những phút ngẫu nhiều, việc đó càng trở nên dễ dàng hơn. Có khi bạn
hứng tình cờ xảy ra làm bạn bất ngờ và làm khán giả còn cảm thấy thích nữa ấy chứ!
thích thú. Bạn sẽ không còn muốn xuất hiện trước
đám đông nếu bạn đã nói về một đề tài cả ngàn lần ƯỚC MƠ CÔ GIÁO
rồi, bạn sẽ cảm thấy chán và chẳng thèm để ý tới
khán giả nữa. Bạn cũng nên lập kế hoạch sẽ mặc Phạm Thị Liên – 51A Toán
những gì. Chú ý rằng đó phải là bộ đồ mà bạn cảm
thấy thoải mái khi mặc vào và điều quan trọng nhất, Tôi đứng đây bên giảng đường rộng mở
đó phải là bộ đồ mà bạn biết sẽ làm mình nổi bật. Tà áo dài chắp cánh những ước mơ
Quyết định trước việc mình sẽ mặc gì trong ngày Nhớ bé thơ cái ngày vừa bập bẹ
diễn thuyết sẽ làm bạn bớt lo lắng hơn. Hãy là chính "Búp bê ngoan nghe cô giáo giảng bài"
mình! Nhiều người cảm thấy cần phải rập khuôn theo Thấm thoắt xưa giờ hết chặng đường dài
phong cách của ai đó khi nói trước đám đông, đó là Nay mười tám tôi không còn khờ dại
vì họ cảm thấy họ không đủ tự tin để lôi cuốn sự chú Khoác áo dài bước tới Đại học Vinh
ý của khán giả. Một số cảm thấy bị “khớp” và Học trò xưa không còn búp bê xinh
nghiêm túc quá mức và quên rằng tính hài hước cũng Bàn ghế xưa cũng không còn gạch đá
là một công cụ quan trọng của diễn giả. Đừng nên chỉ Mà giờ đây lòng tôi đang hối hả
tập trung vào vấn đề chính, đôi khi những giai thoại Mai đến rồi làm cô giáo tương lai.
cá nhân hay những mẩu chuyện nhỏ cũng là một cách
rất tốt để hòa nhập với khán giả. Khán giả là bạn bè!
Khán giả luôn ở đó, bởi vì họ quan tâm tới những gì
bạn sẽ nói và muốn nghe bạn nói về vấn đề đó. Họ
muốn bạn phải làm tốt. Đừng nghĩ khán giả như là
một khối người thù địch, hãy xem họ chỉ là một

TOÁN HỌC & SINH VIÊN số 33


9

TOÁN HỌC TRONG SỰ PHÁT TRIỂN KINH TẾ VI MÔ


MAI THỊ PHƯƠNG – 49A Toán (tổng hợp)

Trong sự phát triển như vũ bão của khoa học kĩ thuật ngày nay cùng với những biến động mạnh mẽ của nền
kinh tế thị trường. Ảnh hưởng của toán học đến các ngành khoa học khác là không hề nhỏ đặc biệt là đối với
nền kinh tế. Thông qua những vấn đề về kinh tế đã được học ở ngành 2 bản thân tôi thấy được sự quan trọng
của toán đối với sự hình thành và phát triển của các mô hình kinh tế. Vì vậy ở đây tôi đề cập đến vấn đề “toán
học trong sự phát triển của nền kinh tế vi mô”. Mà cụ thể hơn là xem xét sự ảnh hưởng của toán học đến hành
vi sản xuất của doanh nghiệp thông qua lý thuyết sản xuất.

I – CÁC KHÁI NIỆM CƠ BẢN VỀ MÔ HÌNH


- Mô hình kinh tế: là mô hình phản ánh các đối tượng trong lĩnh vực hoạt động kinh tế.
- Mô hình toán kinh tế : là mô hình kinh tế được diễn tả bằng ngôn ngữ toán học.
- Bản chất của 1 hệ thống kinh tế là mô hình hóa quá trình vận dụng của nó. Do đó khi xây dựng mô hình toán
học của một hiện tượng kinh tế ta phải chọn các biến: gồm biến ngoại sinh (biến giải thích, biến độc lập) và biến
nội sinh (biến được giải thích, biến phụ thuộc) sau đó tới mô tả quan hệ giữa các biến đó bằng một hệ thức toán
học.
- Các ràng buộc của mô hình: là các hệ thức toán học phản ánh mối quan hệ kinh tế, quan hệ hành vi, quan hệ
mua bán,… giữa các yếu tố kinh tế. Cũng như sự vận động của tự nhiên và xã hội. Quan hệ kinh tế cũng chịu sự
tác động của quy luật bảo toàn (tức là sự bằng nhau, cân bằng theo 1 thước đo nào đó). Mà suy cho cùng hình
thức biểu hiện giữa các biến kinh tế là các phương trình trong toán học.
- Sau khi đưa ra các hệ thức toán học đã xác lập về dạng chính tắc hoặc chuẩn tắc thì ta được phương trình trạng
thái của hệ thống kinh tế đang xem xét.

II – XÂY DỰNG MÔ HÌNH TOÁN KINH TẾ

 Bước 1: Xây dựng mô hình Định tính cho đối tượng kinh tế cấn nghiên cứu. ở đây ta phải xác lập
được quy luật cũng như yếu tố có ý nghĩa quyết định đối với đối tượng kinh tế nghiên cứu.
 Bước 2: Xây dựng mô hình toán học cho đối tượng cần nghiên cứu (tức là diễn tả dưới dạng ngôn
ngữ toán học).
 Bước 3: Sử dụng các công cụ toán học để khảo sát và giải quyết mô hình đã xây dựng ở bước 2
nhằm đưa ra giải pháp tối ưu cho mô hình đã xây dựng.
 Bước 4: Dựa vào các số liệu thu thập được để dự đoán và kiểm định sự phù hợp của mô hình trong
lý luận và thực tiễn, ở đây ta sử dụng mô hình kinh tế vi mô để phân tích cách ứng xử, hành vi của
các chủ thể kinh tế thông qua hành vi sản xuất.

III – CÔNG NGHỆ SẢN XUẤT VÀ HÀM SẢN XUẤT

Hàm sản xuất: Là hàm mô tả những sản lượng sản phẩm đầu ra tối đa có thể được sản xuất bởi một số lượng
yếu tố sản xuất (đầu ra) nhất định tương ứng với trình độ kỹ thuật nhất định. Dạng tổng quát hàm sản xuất

Q = f (X1, X2…, Xn). Trong đó Q là sản phẩm đầu ra, Xi là sản lượng yếu tố sản xuất thứ i.

1) Năng suất trung bình (AP) và năng suất biên (MP)

Năng suất trung bình của một yếu tố sản xuất biến đổi là số sản phẩm sản xuất tính trung bình trên 1 đơn vị yếu
tố sản xuất đó. Năng xuất biên của yếu tố sản xuất biến đổi là phần thay đổi trong tổng sản lượng khi thay đổi 1
đơn vị yếu tố sản xuất biến đổi đó.

Q
Cho hàm sản xuất Q = f (X1, X2,…, Xn ). Gọi năng suất trung bình theo 1 yếu tố sản xuất thứ i là hàm
Xi

TOÁN HỌC & SINH VIÊN số 33


10
'
Q X i f X' i ( X )  f ( X ) f (X ) Q
(i= 1, n ) có cực đại tại Xi*. Ta có   = 2
 0. Do vậy, f x'i ( X )   .
 X i  X i  X i* Xi Xi Xi

2) Tiến bộ kĩ thuật

 y  R n
Giả sử công nghệ sản xuất của một doanh nghiệp được biểu thị bởi hàm sản xuất y = f(X)  Khi đó
n
 x  R .
X
aij  i là hệ số kĩ thuật của quá trình sản xuất. Giả sử ta có công nghệ sản xuất biểu thị dưới dạng hàm sản
Yj
xuất Q  A(t ). f (k , l ) 1 trong đó k là vốn, l là sức lao động, A(t) là tất cả các yếu tố ảnh hưởng khác (là hàm
d ( A(t ))
của thời gian t). Nó biểu thị tiến bộ kĩ thuật cho nên ta luôn giả sử là  0. Khi đó vi phân 2 vế của (1)
dt
dQ d ( A(t )) f dk f dl dQ
theo rút gọn ta được:   .  .  2  . Đặt  GQ là tốc độ tăng trưởng
Qdt A(t ).dt f ( k , l ).k dt f (k , l )l dt Qdt
d ( A(t )) dk
của Q theo thời gian;  G A là tốc độ tăng trưởng của A theo thời gian;  GK là nhịp tăng của vốn
A(t ).dt kdt
dl
theo thời gian và  GL là tốc độ tăng trưởng của lao động theo thời gian. Khi đó (2) có thể viết thành:
ldt
GQ  GA  E (Q, k ).Gk  E (Q, l ).Gl . Như vậy, nhịp tăng của sản lượng được biểu diễn qua 3 thành phần
G A , Gk , Gl suy ra nhịp tăng của sản lượng Q là do phần đóng góp của vốn, của lao động, phần của tiến bộ kĩ
thuật.

*************************************************************************************
ĐẠI HỌC VINH – NƠI TẠO DỰNG TƯƠNG LAI CHO TUỔI TRẺ
TRẦN THỊ THANH NHÀN – 48A Toán

Trong mỗi chúng ta, chắc hẳn ai cũng còn nhớ trưởng thành của Khoa Toán qua 50 năm xây dựng
cảm giác bỡ ngỡ, niềm vui tựu trường xen lẫn và phát triển. Cùng nhìn lại những hoạt động và kết
những lo lắng trong buổi đầu nhập học. Lúc ấy, quả đạt được, sinh viên Khoa Toán không khỏi
hành trang mang trên mình chỉ là những ước mơ, không tự hào về những thành tích đó. Bên cạnh
những hoài bão, là cả một tuổi trẻ với trọn vẹn bầu những thành tích về các hoạt động Đoàn Thanh
nhiệt huyết. Điều đó cũng dễ hiểu. Bởi lẽ, là những niên, Hội sinh viên thì phong trào học tập, nghiên
người trẻ tuổi, có ai mà không ôm một giấc mơ cứu khoa học (NCKH) của Khoa cũng có nhiều
trong vòng tay, trong trái tim hay trong tâm tưởng bước tiến, luôn là hoạt động bề nổi và dẫn đầu.
của mình! Và trong số các bạn, có ai mà chưa một + Về học tập
lần mơ ước rằng, mai này, mình sẽ trở thành một Khắc phục được những khó khăn ban đầu về
nhà giáo, một nhà khoa học hay một kỹ sư! Đó là lý chương trình học theo hệ thống tín chỉ, cùng với sự
do để tất cả chúng ta về đây, dưới mái trường Đại giúp đỡ, tạo điều kiện của Khoa và Nhà trường, mỗi
học Vinh này, để cùng một mục đích là thực hiện sinh viên Khoa Toán đã dần làm quen được với hình
những ước mơ, dự định đó. Chính mái trường Đại thức đào tạo mới, có nhiều cố gắng trong quá trình
học Vinh là ngôi nhà chung để tất cả chúng ta giao học tập, rèn luyện của mình.
lưu, học hỏi, tự bồi dưỡng và giúp đỡ lẫn nhau trong Là một sinh viên Khóa 48- khóa đầu tiên đào tạo
học tập, rèn luyện. Và nơi đây cũng là nơi ươm theo hệ thống tín chỉ, ý thức được những khó khăn
mầm những ước mơ, nơi tạo dựng tương lai cho tuổi của cách đào tạo mới, cũng như các sinh viên khóa
trẻ. 48 khác, mình đã phải cố gắng rất nhiều để xây
Hòa cùng với niềm vui chung của cả trường, dựng cho bản thân một phương pháp học tập phù
năm học qua cũng là mốc thời gian đánh dấu sự hợp, cụ thể như:

TOÁN HỌC & SINH VIÊN số 33


11
- Lập cho mình một thời khóa biểu, thời gian - Tham gia các buổi tọa đàm, seminar bàn về
biểu hợp lý. Đặc biệt là đối với việc đăng ký học, phương pháp học tập, trao đổi kinh nghiệm.
các bạn nên đăng ký các môn học theo khung - Luôn tạo cho mình thói quen tự học, tự tìm tòi,
chương trình đào tạo của Khoa. Trong quá trình nghiên cứu.
đăng ký, nên chú ý cân đối giữa các học phần lý Bên cạnh những vấn đề đã nêu trên, còn một điều
thuyết và các môn có nhiều bài tập. Sắp xếp lịch học mà mình nghĩ rằng nó cũng không kém phần quan
một cách phù hợp, sao cho tiết kiệm được thời gian trọng. Đó là chúng ta phải không ngừng vun đắp
nhất. những ước mơ, hoài bão, lý tưởng cao đẹp, từ đó mới
- Luôn có kế hoạch học tập cụ thể, lựa chọn và có được động cơ, thái độ và phương pháp đúng đắn
xây dựng phương pháp học thích hợp, có thể phối trong học tập và NCKH.
hợp nhiều phương pháp học khác nhau để kết quả tự Như văn hào Lép-tôn-xtôi đã từng nói: “Lý tưởng
học đạt tối ưu. là ngọn đèn chỉ đường, không có lý tưởng thì không
- Trong quá trình học, cần nắm vững nguyên tắc: có phương hướng kiên định, mà không có phương
Tìm hiểu bất cứ vấn đề gì bao giờ cũng bắt đầu đi từ hướng thì không có cuộc sống”…Vâng, mình thiết
dễ đến khó, không đặt mục tiêu quá cao đối với năng nghĩ rằng, con đường hôm qua, hôm kia của mình,
lực bản thân mình. của bạn sẽ dần lùi vào quá khứ. Nhưng con đường
- Luôn có ý thức nỗ lực ý chí, kiên trì khắc phục hôm nay và của ngày mai còn tùy thuộc vào tất cả
khó khăn trong quá trình tự học, rèn luyện thói quen chúng ta. Chúng ta phải đi như thế nào, chọn lựa
độc lập suy nghĩ, chủ động sáng tạo trong mọi vấn ngọn đèn lý tưởng nào và đi theo phương hướng ra
đề. sao để tiếp tục phát triển, đó mới là điều quan trọng.
Như chúng ta đã biết, học Đại học là biến quá Hy vọng rằng, với bầu nhiệt huyết sẵn có, niềm say
trình đào tạo thành quá trình tự đào tạo. Do đó, theo mê khoa học, tuổi trẻ Khoa Toán sẽ tiếp tục học tập,
mình, mỗi người cần xây dựng kỹ năng tự học cho nghiên cứu và sẵn sàng cống hiến, xứng đáng là niềm
mình. Ở lớp, chú ý nghe giảng và ghi chép đầy đủ, cố tự hào của Khoa Toán, xứng đáng là những sinh viên
gắng hiểu những vấn đề được thầy cô nói tới trong trường Đại học Vinh “Bản lĩnh, Trí tuệ, Văn minh,
bài giảng và trọng tâm của bài học. Về nhà, xem lại Tình nguyện”.
bài và làm bài tập để có thể nhớ và vận dụng các kiến
thức đã học, thành thạo các kỹ năng giải các dạng bài ẢNH VUI TOÁN HỌC (!)
tập. Chỗ nào chưa hiểu thì có thể cùng trao đổi với
bạn bè hay hỏi lại thầy cô.
- Nên học các môn ngay từ đầu và học một cách
thực sự chứ không nên để đến lúc ôn thi mới bắt đầu
học. Như vậy kiến thức sẽ không nhớ được lâu. Đối
với các môn lý thuyết, các bạn nên đọc giáo trình và
tập cách lập đề cương để học. Và cũng tương tự, nên
hệ thống kiến thức và phân dạng cùng cách giải các
dạng đối với các môn có nhiều bài tập.
Mình nghĩ rằng, nếu các bạn chịu khó tìm cho
mình một phương pháp học tập đúng đắn, phù hợp
với bản thân thì việc tự học sẽ diễn ra thuận lợi hơn.
+ Về công tác NCKH
NCKH là một hoạt động quan trọng trong việc
nâng cao chất lượng đào tạo trong các trường đại học.
Theo mình, để có thể NCKH, mỗi sinh viên chúng ta CÁC BẠN ĐƯỢC THƯỞNG KỲ NÀY
cần tham gia các buổi Hội nghị phương pháp học tập
do Khoa tổ chức để dần làm quen với việc NCKH. TOÁN HỌC SINH VIÊN SỐ 33 (11/2010)
Bởi thông qua các báo cáo điển hình về phương pháp 1. Đỗ Đức Hiếu, SV lớp Anh3-CLC-KT-K49, ĐH
học tập các môn chuyên ngành ở Đại học, chúng ta Ngoại Thương, Hà Nội,
bước đầu được làm quen với cách đặt vấn đề, triển 2. Phan Minh Trí, HS lớp 12T, trường THPT chuyên
khai ý tưởng của mình về một đề tài ở bộ môn yêu Lương Văn Chánh, Phú Yên.
thích. 3. Phạm Thị Liên, SV lớp 51A Toán, ĐH Vinh.
- Tham gia các cuộc thi Olympic Toán, viết bài 4. Đinh Bích Yến, SV lớp 49A Toán, ĐH Vinh.
cho tập san Toán học và Sinh viên.

TOÁN HỌC & SINH VIÊN số 33


12

NHỮNG ĐỊNH LÝ GIẢI TÍCH QUAN TRỌNG


TRẦN QUỐC LUẬT – 50A Toán, NGUYỄN THANH HUYỀN – 51A Toán

Tóm tắt. Nhằm giúp các bạn sinh viên khoa Toán Đại học Vinh tiếp cận và hiểu hơn về lí thuyết giải tích
cổ điển chúng tôi sẽ đưa ra các định lí quan trọng trong giải tích toán học và những ứng dụng của nó trong
giải tích cổ điển.

A – TÓM TẮT LÝ THUYẾT với mọi x  (a, b ) thì f là hàm hằng trên [a , b].
Để mở đầu chúng tôi xin nhắc lại các Định lí Hệ quả 1.6: Nếu f liên tục trên [a , b] và f ( x)  0
Rolle, Lagrange và Cauchy.
với mọi x  (a, b ) thì f là tăng ngặt trên [a , b].
Định lí Rolle: Giả sử f là hàm liên tục trên [a,b] ,
khả vi trên (a, b) và f (a )  f (b) thì tồn tại
B – CÁC DẠNG BÀI TẬP ÁP DỤNG ĐỊNH LÝ
c  ( a, b ) sao cho f (c )  0. LAGRANGE
Hệ quả 1.1: Nếu f là hàm khả vi trên R thì giữa Ví dụ 1:
bất kì hai nghiệm nào của f cũng tồn tại một Giả sử a0 , a1 ,..., an là các số thực thỏa mãn:
nghiệm của f '. a0 a a
 1  ...  n 1  an  0.
Hệ quả 1.2: Nếu f là hàm khả vi trên R và n 1 n 2
f ( x)  0 với mọi x thì f có nhiều nhất một Khi đó đa thức P ( x)  a0 x  a1 x n 1  ...  an có ít
n

nghiệm. nhất một nghiệm trên đoạn (0,1).


Hệ quả 1.3: Cho n  N , giả sử f là hàm khả vi a a
Giải: Xét Q( x)  0 x n1  1 x n  ...  an x.
cấp n (tức là tồn tại f ( n ) ( x) với mọi x ). Nếu n 1 n
Ta thấy Q( x) thỏa mãn điều kiện Định lí Rolle
f ( n ) ( x)  0 với mọi x thì f có tối đa n nghiệm.
Từ Định lí Rolle ta đi chứng minh Định lí Cauchy Q(0)  Q(1) nên P( x)  Q( x)  0 có ít nhất một
và Lagrange. nghiệm trên (0,1). (đpcm)
Như chúng ta đã biết khi chứng minh Định lí Ví dụ 2:
Lagrange để áp dụng Định lí Rolle hầu hết các giáo Chứng minh nếu hàm f có đạo hàm cấp 2 trên
trình đều chọn hàm phụ có dạng đoạn [a,b] và f ( a )  f (b)  0 thì bất phương trình
f (a )  f (b)
F ( x )  f ( x )  f (a )  ( x  a ). 4
ba f ( x )  f (b )  f ( a )
(b  a ) 2
Ta nhận thấy F ( a )  F (b)  0 và từ đó theo Định
có ít nhất một nghiệm.
lí Rolle ta có tồn tại c   a; b  để F (c )  0. Giải: Nếu f ( x)  const thì bài toán hiển nhiên
Định lý Lagrange: Giả sử f là hàm liên tục trên đúng.
[a,b] , khả vi trên (a, b) thì tồn tại c  (a,b) sao cho Ngoài ra áp dụng Định lí Cauchy với hàm f và
f (b )  f ( a ) ( x  a)2 ab
f (c)  . hàm   x  [ a, ] ta được:
ba 2 2
Định lí Cauchy: Cho hai hàm f , g :[a, b]  R là  a b 
8 f ( )  f (a ) 
các hàm liên tục trên [a, b] và khả vi trên (a , b)  2   f (1 ) ,  a    a  b  .
1  a  
1
đồng thời g ( x)  0 với mọi x thì tồn tại c  ( a, b ) (b  a) 2 2 
f (b)  f (a) f (c) Tương tự áp dụng Đính lí Cauchy cho hàm f và
sao cho  .
g (b)  g (a) g (c) (b  x) 2 ab
hàm   x [ , b ] ta được:
Tiếp theo chúng ta sẽ nói về ứng dụng của hai 2 2
định lí này, có thể nói hai định lí này là một tài sản  a b 
quí báu của những ai học toán, những ứng dụng của 8  f (b)  f ( )
 2  f ( 2 )  a  b 
nó liên quan đến hội tụ đều, tìm giới hạn, số nghiệm  ,  2  b .
của phương trình, chứng minh bất đẳng thức,.... (b  a) 2 b  2  2 
Hệ quả 1.4: Giả sử f là hàm khả vi trên khoảng I , Cộng hai đẳng thức ở trên ta có:
f ' bị chặn trên I thì f là hàm liên tục đều trên I . 8  f (b )  f (a )  f (1 ) f ( 2 )
  .
Hệ quả 1.5: Nếu f liên tục trên [a , b] và f ( x)  0 (b  a ) 2 1  a b   2

TOÁN HỌC & SINH VIÊN số 33


13
Theo giả thiết, ta có f ( a )  f (b )  0 nên ta áp Ví dụ 4:
dụng Định lí Lagange ở trên và viết lại đẳng thức Cho số thực a  2 và đa thức
trên dưới dạng: f n ( x)  a10 x n 10  x n  ...  x  1 .
f (1 ) f ( 2 ) f (1 )  f (a ) Chứng minh rằng với mỗi số nguyên dương n ,
 
1  a b   2 1  a phương trình f n ( x)  a luôn có đúng một nghiệm
f (b )  f ( 2 ) dương duy nhất. Gọi nghiệm đó là xn , chứng minh
  f (1 )  f (2 ),
b  2 rằng dãy xn có giới hạn hữu hạn khi n dần đến vô
trong đó a  1  1 , 2   2  b. Từ đó ta rút ra cùng.
được: Giải: Dễ dàng chứng minh tính tồn tại và duy nhất
a 1
8  f (b)  f (a )  của xn . Chúng ta sẽ chứng minh lim xn  .
 f (1 )  f (2 )  n a
(b  a )2 a 1
Thật vậy, đặt c   1 , khi đó ta có:
 2 max  f (1 ) , f ( 2 )   2 f (c) . a
4 f n (c)  f n ( xn )  kc n
Do đó ta có f (c)  f (b)  f (a ) . (đpcm)
(b  a ) 2
Ví dụ 3:
 9

(với k   a  1  a  1  1  0 ).

Cho hàm f khả vi trên ( x0 , ) và lim f ( x)  0. Theo Định lý Lagrange thì:
x
f n (c )  f n ( xn )  f ( )(c  xn ) với    xn , c  .
f ( x)
Chứng minh lim  0. Nhưng f ( )  1 nên từ đây suy ra: kc n  c  xn . Từ
xx
Giải: Giả sử ( xn ) là dãy tuỳ ý sao cho xn   . đó ta có : c  kc n  xn  c.
 a 1
Khi đó   0, N : n  N ta có f ( xn )  . Vậy ta có lim xn  c  . (đpcm)
2 n  a
Cố Định n0  N và lấy n  n0 , áp dụng Định lý
Ví dụ 5:
f ( xn )  f ( xn0 ) Cho n là một số nguyên dương. Chứng minh rằng
Lagrange:  f ( nn0 ) , trong đó
xn  xn0 1 1 1 1
phương trình   ...  2  có
xn0   nn0  xn . Do bất đẳng thức trên nên ta có x  1 4x  1 n x 1 2
một nghiệm duy nhất xn  1 . Chứng minh rằng khi
f ( xn )  f ( xn0 ) 
 . n dần đến vô cùng, xn dần đến 4.
xn  xn0 2 1 1 1 1
Giải: Đặt f n ( x)    ...  2  .
Ta biến đổi bất đẳng thức trên và thu được: x  1 4x  1 n x 1 2
f ( xn0 )  xn   f ( xn ) f ( xn0 )  xn0   Gọi xn  1 là nghiệm duy nhất của phương trình
 1  0     1   .
xn  xn  2 xn xn  xn  2 f n ( x)  0 . Ta có
1 1 1 1
f n (4)    ...  2 
Cho n tiến ra vô cùng, do dãy ( xn ) đã cho ở trên 4  1 16  1 4n  1 2
 f ( xn0 )  1 1 1 1
nên ta có     . Mặt khác ta có    ...  
2 xn 2 1.3 3.5 (2n  1)(2n  1) 2
 xn0    1 1 1 1 1 1 1  1
      ...   
1    với mọi n  n0 . Từ hai nhân xét 2 1 3 3 5 2n  1 2n  2
 xn  2 2
1
f ( xn ) f ( xn )  .
trên ta thu được     hay  . 4n
xn xn Áp dụng Định lý Lagrange, ta có :
f ( x) 1
Do đó ta có lim  0 (đpcm).  f n ( xn )  f (4)  f (c) xn  4 , c   xn , 4  .
x  x 4n
Chúng ta lại xem xét tiếp, khi tính giới hạn của 1 4 1
các hàm đặc biệt thì các phương pháp cổ điển hầu Nhưng do | f n '(c ) | 2
 2
 ...  .
(c  1) (4c  1) 9
như đều quá khó để dùng, đôi lúc ta dùng qui tắc
L'Hospital tuy nhiên không phải lúc nào nó cũng là 9
nên từ đây xn  4  , suy ra lim xn  4. (đpcm)
hiệu quả lúc đó ta áp dụng Định lý Lagrange. 4n n 

TOÁN HỌC & SINH VIÊN số 33


14

Ví dụ 6: ln  k  1 ln 2  k  1 ln 2 k ln k
Chứng minh Bất đẳng thức Bernoulli:    .
n k 1 2 2 k
Nếu x  1 thì 1  x   1  nx với mọi n  N . Từ bất đẳng thức trên, ta có:
Giải: Giả sử x  0 và xét f (t )  (1  t )n với ln( n  1) ln 2 (n  1) ln 2 n
xn1  xn    0
t  [0, x] . Theo Định lí Lagrange ta có tồn tại n 1 2 2
  [0, x] sao cho f ( x)  f (0)  ( x  0) f ( ) hay nên ta có dãy xn giảm. Dễ thấy tính bị chặn là hiển
n n 1 nhiên nên dãy xn có giới hạn hữu hạn.
1  x 
 1  xn 1     nx.
Ví dụ 11:
Trường hợp 1  x  0 ta làm tương tự.
Cho dãy số thực u n  được xác Định như sau
Ví dụ 7: 1
Cho hai số thực dương a  b. Khi đó ta có: u1  a  R, un1 
2
 
ln 1  un2  2010, n  1.
ba b ba
 ln  . Chứng minh u n  là một dãy hội tụ.
b a a
1
Giải: Xét hàm f (t )  ln t , t [ a, b] . Theo Định lí Giải: Ta có f ( x) 
2
 
ln 1  x 2  2010 là hàm liên
Lagrange thì tồn tại    a; b  sao cho:
x 1
b 1 tục trên R và f ( x)   , x  R. Mặt
f (b)  f ( a )  (b  a ) f ( ) hay ln  b  a  . 1 x 2
2
a 
1
Do    a; b  nên ta có đpcm. 
khác đặt g ( x)  x  2010  ln 1  x 2  x  f ( x)
2

Ví dụ 8: thì g ( x) cũng là hàm liên tục trên R và
Chứng minh bất đẳng thức x2  x  1
g ( x )   0. Như vậy g ( x) là hàm liên
n
n 1 n
x2  1
  k  1   k  trong đó   0, n  N .
 tục và đơn điệu tăng trên R. Mặt khác, do
k 1   1 k 1
g (0) g (2002)  0 nên phương trình f ( x)  0 có
t  1
Giải: Xét hàm f (t )  với t  [k  1, k ] nghiệm duy nhất, ta gọi nó là L . Theo Định lí
 1
Lagrange, tồn tại   R sao cho:
Theo Định lí Lagrange ta có tồn tại    k  1, k 
un 1  L  f (u n )  f ( L)  f ( ) u n  L .
sao cho f ( k )  f (k  1)  f ( ). Do đó: n 1
 1 1 1
k  1

 k  1   .
Suy ra 0  un  L  un 1  L  ...    u1  L .
2  2
 1  1
 1 Từ đó ta có lim un  L.
 k  1  k  1 n
Từ đó suy ra  k  1    k  do Ví dụ 12:
 1  1 n
   k  1, k  .  1
Chứng minh rằng Sn  1   là dãy tăng còn
Cho k chạy từ 1 đến n và cộng n bất đẳng thức  n
n 1
trên ta thu được bất đẳng thức cần chứng minh.  1
Tn   1   là dãy giảm.
Ví dụ 9:  n
n
ln k ln 2 n Giải: Xét hàm f ( x)  x n . Theo Định lý Lagrange,
Chứng minh dãy xn  
k 1 k

2
hội tụ và có
bn  a n
ta có:  nc n 1 , (a  c  b).
giới hạn hữu hạn. ba
ln 2 t Do na n1  nc n 1  nb n1 nên
Giải: Ta xét hàm f (t )  với t  ( k , k  1).
2 bn  an
Theo Định lý Lagrange, tồn tại   ( k , k  1) sao na n 1   nb n 1 .
ba
ln 2  k  1ln 2 k ln   n 
1/ n
cho   . Ta chọn a  1, b    , khi đó
2 2   n 1 
Mặt khác ta dễ dàng kiểm tra được rằng hàm n 1 n
 n   n 1 
f (t ) 
ln t
đơn điệu giảm trên [3, ) nên ta có     hay S n là dãy tăng.
 n 1   n 
t

TOÁN HỌC & SINH VIÊN số 33


15

 n2
1/ n Mời các bạn hãy xem các tổ hợp 3 số dưới đây,
Tương tự ta chọn b  1, a    ta cũng sẽ thu
 n 1  mỗi số có 6 chữ số. Chia các tổ hợp thành hai nhóm,
được Tn là dãy giảm. tổng của các chữ số trong hai nhóm bằng nhau. Ví
Ví dụ 13:
dụ:
Cho hàm f ( x) khả vi trên [a, b] và thỏa mãn
f ( x)  0 có nghiệm trên [a, b ], đồng thời 123789  561945  642864  242868  323787  761943.
f '( x)  f ( x) với x [a , b]. Chứng minh rằng Tính chất vừa nêu không có gì lạ có nhiều tổ hợp
f ( x)  0 với mọi x thuộc [a, b ]. số cũng có tính chất đó. Nhưng nếu chú ý thì sẽ thấy
Giải: Gọi x0 là nghiệm của f ( x). Đặt
các tổng bình phương các số trong nhóm:
g ( x)  f ( x)e x . Dễ thấy g ( x) tăng. Ta có g ( x)  0
1237892  5619452  6428642  2428682  3237872  7619432.
và g ( x0 )  0. Do đó f ( x)  0, x [ a, x0 ].
Bạn đừng tán thưởng vội, đó mới chỉ là khúc dạo
Đặt h( x)  f ( x)e x . Dễ thấy h ( x) giảm. Ta có
đầu. Bây giờ hãy tước bỏ các chữ số ở đầu mỗi con
h ( x)  0 và h ( x0 )  0 . Do đó f ( x)  0, x  [ x0 , b ].
Vậy f ( x)  0. số các bạn sẽ thấy điều thần diệu của các con số có
Trên đây là những dạng toán áp dụng Định lý 5 chữ số vừa mới tạo thành:
Lagrange, trong số báo sau chúng tôi sẽ giới 23789  61945  42864  42868  23787  61943.
thiệu kỹ về Định lý Rolle và Công thức Taylor.
237892  619452  428642  428682  237872  619432.
Mời các bạn giải các bài tập vận dụng Định
lý Lagrange: Quả kì lạ(!) Bạn lại tước bỏ các chữ số đứng ở đầu
n n 1
1. Chứng minh 1  1 / n   e  1  1 / n  các con số vừa mới hình thành bạn sẽ có một bộ các
với mọi n nguyên dương. số, mỗi số có 4 chữ số. Qua tính toán bạn sẽ thấy:
2. Chứng minh
3789  1945  2864  2868  3787  1943.
 1 1 1  ln(1  a )
lim    ...   . 37892  19452  28642  28682  3787 2  19432.
n   n  a n  2a n  na  a
Bây giờ chúng ta lại tiếp tục công việc đang dở
3. Cho 0  a  b. Chứng minh:
dang, lại tiếp tục bỏ các chữ số ở đầu mỗi con số,
a b ab
ab   . rồi tính toán, lần lượt ta sẽ có các đẳng thức kì diệu:
ln a  ln b 2
4. Chứng minh nếu hàm f liên tục trên 789  945  864  868  787  943.
[a, b ], có đạo hàm cấp 2 trên đoạn [a,b] và 789 2  945 2  864 2  868 2  787 2  94 32.
f ( a )  f (b )  0 thì bất phương trình
89  45  64  68  87  43.
f (b )  f ( a )
f ( x )  có nghiệm. 892  452  642  682  87 2  432.
ba
5. Chứng minh Và màn “thoát y” cuối cùng đây:
 1  2  ...  n  1 9  5  4  8  7  3.
lim   .
n 
 n 1   1 92  52  4 2  82  7 2  32.
HIỆN TƯỢNG "THOÁT Y VŨ" Bây giờ ta lại làm việc ngược lại là tước bỏ lần lượt
TRONG TOÁN HỌC các chữ số cuối của mỗi con số ta cũng lại sẽ có
ĐINH BÍCH YẾN – 49A Toán
những điều kì diệu khác. Ví dụ:
Nói về "thoát y vũ" trong Toán học, trước hết
12378  56194  642 86  24286  32378  76194.
phải kể đến đối tượng có thể thoát y là cái gì (bởi 12378 2  56194 2  64286 2  24286 2  32378 2  7619 4 2.
không phải đối tượng nào muốn thoát y là được?). Cuối cùng ta cũng có:
Đầu tiên phải kể đến là chữ số. Chữ số thoát y vũ là 1  5  6  2  3  7.
nói về tổ hợp các số có nhiều chữ số. Khi tước bỏ 12  5 2  6 2  2 2  3 2  7 2 .

từng vị trí thì nó biến đổi ra sao? Bạn xem có kì lạ không!

TOÁN HỌC & SINH VIÊN số 33


16

ĐỊNH LƯỢNG TOÁN TỬ TUYẾN TÍNH


TRẦN QUỐC LUẬT – 50A Toán

Tóm tắt. Bài viết tập hợp một số bài toán định lượng toán tử tuyến tính nhằm giúp các bạn sinh viên năm
thứ ba có một cái nhìn toàn diện hơn về toán tử tuyến tính và làm quen với loại bài tập này. Trong bài viết
này, nếu không chú thích gì thêm thì chuẩn trên không gian cho trước là chuẩn thông thường trên không
gian đó.

A – PHƯƠNG PHÁP b

Bước 1: Xác Định không gian định chuẩn và chuẩn tích phân f   f ( x) dx.
1
của nó là chuẩn gì. a

Bước 2: Tìm hằng số k thỏa mãn f ( x)  k . x . Ta thử tính chuẩn f với chuẩn tích phân này:
b
Bước 3: Chọn x0 sao cho x0  1 và f  k  .
Ta có: f ( x )  f ( x)  eb x(t ) dt  eb x . Do đó 
1
Ta sẽ làm rõ điều này thông qua một số ví dụ từ dễ a
đến khó. b
f  e . Với mọi a  c  d  b, xét hàm x(t )  0
Ví dụ 1: Cho T : C[0;1]  C[0;1] biến x thành T ( x)
xd
với (T ( x ))(t )  et x(t ), t  [0;1] . Chứng minh T với a  t  c, x(t )  1  với
d c
tuyến tính, liên tục và tính chuẩn T . c  t  d và x(t )  1 với d  t  b . Ta có
Giải: Với mọi x, y  C[0;1] ;  ,   R ta có: d c x
x   b  d . Xét hàm y  khi đó
1 2
(T ( x   y ))(t )  et  x   y  (t )  x
1
t t
 .e x(t )   .e y (t )   .T ( x)  (t )   .T ( y )  (t ). y  1 và
1
Vậy T là ánh xạ tuyến tính. Ta có: b dt 2b  d 
f ( y)  f ( y)  ed   ed .
Tx  sup et x (t )  sup ( x(t )). sup et  e x . d x 2   d  d c
b
t[0,1] t[0,1] t[0,1] 1

Tx Cho c tiến đến d , ta có f  e d với mọi


Suy ra T liên tục và T  sup  e.
x0 x a  d  b. Cho d tiến đến b ta được f  eb .

Với x0 (t )  1, với mọi t  R thì x0  1 và 1
Ví dụ 3: Cho f : C0   với f ( x)  2 n
xn với
n 1
Tx0
T   e. Vậy T  e. mọi x  {xn }  C0 . Tính chuẩn f .
x0

1
Chú ý: Ta có thể thay et bởi một hàm g (t ) bất kỳ bị Giải: Ta có f ( x )  f ( x)  2
n 1
n
x  x . Do đó
chặn trên [0;1]. Khi đó bằng chứng minh tương tự,
f  1. Với mọi   0 , tồn tại N sao cho
ta có T  sup g (t ).
[0;1] N
1
Ví dụ 2: Cho f : C[ a ,b ]  R xác định bởi 2 n
 1   , chọn dãy x ( N )  ( xn( N ) ) như sau
n 1
b
f ( x)  
t
e x(t ) dt , x  C[ a,b] . Chứng minh f tuyến xn( N )  1 nếu n  N và xn( N )  0 nếu n  N . Khi
a

tính, liên tục. Tính chuẩn f . đó x ( N )  C0 và x( N )  1 và


Giải: Tính tuyến tính là hiển nhiên. Ta có N
1
f  f ( x( N )  2  1   với mọi   0,
f ( x)  f ( x)  (eb  e a ) x nên f  eb  e a . n 1
n

Chọn x0 (t )  1, t  R thì x0 (t )  C[ a ,b ] . Ta có do đó f  1. Vậy f  1.


b a b a
f ( x )  e  e . Do đó: f  e  e . Ví dụ 4: Cho f : l  l1 xác định
Chú ý rằng trên C[ a,b] có chuẩn thông thường là 1
f  xn    n 2 xn  mọi x  {xn } . Tính chuẩn f .
chuẩn sup như trên nhưng ngoài ra còn có chuẩn  

TOÁN HỌC & SINH VIÊN số 33


17
  1  1
Giải: Ta có f ( x)  f ( x )   x . Do đó A  sup Ax  A( x0 )  max Ax0 (t )  Ax0    2.
2 
 n1 n  x 1 [0;1]  2
 Vậy A  2.
1
f  
n 1 n 2
. Chọn x0 là dãy số đồng nhất bằng 1,
Ví dụ 8: Giả sử C[0,1] là không gian tuyến tính các
khi đó x0  l , x0  1, f ( x0 )  l1 và hàm số liên tục trên [0;1], với chuẩn sup. Ánh xạ

1 2 X : C[0;1]  C[0;1] xác định như sau:
f  f ( x0 ). Do vậy f  
n 1 n
2

6
.
Ax(t )  x(1)  tx(t ) , t  [0;1], x  C[0;1] . Chứng minh
A tuyến tính liên tục và tính chuẩn của A.
Ví dụ 5: Cho ánh xạ f : C0  l1 với Giải: Tương tự như trên ta có A tuyến tính liên tục
x  và A  2. Với mỗi n  N * , đặt:
f ( x)   nn  , x  { xn }  C0 . Tính chuẩn f .
3   n 
Giải: Ta có x0 (t )  1  0  t  ;
 n 1
 
xn 1 1
f ( x)  
n 1 3
n
 sup
n 1
xn .n 1 3
n
 x . Do đó
2
 n
x0 (t )  (2n  1)t  2 n  1 

 t  1  . Khi đó
 n 1 
1
f  . Chọn x n  (1,1,....,1,0,0,...), trong đó n số x0 (t )  C[0;1] và x0  1. Ta có:
2
hạng đầu bằng 1, các số hạng sau bằng 0, thế thì A  sup A( x)  max A( x0 )(t )
x 1 [0;1]
n
1 1
x n  c0 và f  f ( x n )   k
 khi n  .  n  2n  1
k 1 3 2  A( x0 ).   2, n  .
 n 1 n 1
1
Như vậy f  . Vậy A  2.
2
Ví dụ 8: Gọi C là không gian các hàm x (t ) liên
Ví dụ 6: Cho ánh xạ f : l1  l1 xác định
tục trên [0, ) thỏa mãn sup e t | x (t ) | . Xét
 n  [0;  )
f ( x)   xn  , x  { xn }  l1 . t
 n 1  chuẩn x

 sup e | x(t ) | và xét ánh xạ
[0; )
Tính chuẩn f .
B : C  C   xác định B( x(t ))  b(t ) x(t ) với
 
n b(t )  C ,   0. Chứng tỏ B liên tục và tính
Giải: Ta có f ( x )  n 1 n 1
.xn  x
n 1
n  x.
chuẩn của B.
Chọn dãy x n  (0,0,...,1,0,...) trong đó số 1 đứng ở Giải: Ta có
n Bx  sup e()t b(t)x(t)  sup et x(t). sup et b(t)  x . b
vị trí thứ n. Ta có: f ( x n )  . 0,  0, 0,
n 1
n Ta lại có B  sup Bx  Bx . Chọn x0 (t )  e t ,
Do đó f  f ( x n )  . x 1
n 1
Cho n  , ta có f  1. Vậy f  1. thì x0  1 và Bx0  b . Vậy B  b .
Sau đây là một số bài tập vận dụng:
Ví dụ 7: Giả sử X  {x  C[0,1] : x(0)  x(1)  0}. Xét 1. Giả sử C[0,1] là không gian tuyến tính các hàm số
ánh xạ T : X  X xác định bởi công thức liên tục trên [0;1], X  { f C[0,1] : f (0)   f (1)}.
Ax(t )  x(t )  x(1  t ), t  [0,1], x  X . Chứng minh Xét hàm T : X  C[0,1] với
A tuyến tính liên tục và tính chuẩn của A.
T ( f )( x)  x3 . f ( x ), x  [0,1], f  X .
Giải: Tương tự như trên ta có A tuyến tính liên tục
Tính chuẩn của T .
và A  2. Xét hàm số:
2. Giả sử X  { x  C[0,1] : x(0)  x(1)  0}. Xét ánh
 1 1 
x0 (t )  2t  0  t   ; x0 (t )  2t  2   t  1  xạ T : X  X xác định bởi công thức
 2 2  Ax(t )  x(t )  x(1  t ), t  [0,1], x  X . Chứng minh
Khi đó x0 (t )  X và x0  1. Ta có: A tuyến tính liên tục và tính chuẩn của A.

TOÁN HỌC & SINH VIÊN số 33


18

ĐỀ RA KỲ NÀY
Bài T1/33: Giải phương trình: f  x   1  x n  x 2 n  ...  x 
m 1 n
chia hết cho đa
4 x  x  1  log3 (8 x  1).
thức g  x   1  x  x 2  ...  x m 1 khi và chỉ khi m
Nguyễn Tất Thu – (cựu SV 40A2 Toán)
Bài T2/33: Tính tích phân: và n nguyên tố cùng nhau.
 Phan Minh Trí – TP. Tuy Hòa – Phú Yên
3
x 2 dx Bài T12/33: Cho p là số nguyên tố. Giải phương
I 
( x sin x  cos x) 2
o
.
trình sau trên tập các số tự nhiên: a p  1  p k .
Phạm Kim Chung – (cựu SV 41A1 Toán) Đỗ Chí Sơn Linh – 12A1 – Trường Chuyên ĐHV
Bài T3/33: Giải phương trình:
Bài T13/33: Cho hàm f :  0;1  R liên tục, thỏa
cos 2 x  cos 6 x  3 sin x cos x. 1
Trần Quốc Luật – 50A Toán minh tồn tại c   0;1 để
Bài T4/33: Tìm giá trị lớn nhất của biểu thức:
mãn  xf ( x)  0. Chứng
0
2

P
 tan 2010
x  cot 2010 x   2 2010
.
1

 tan 2
x  cot 2 x
2011 
f (c )  c f ( x) dx.
c

Đậu Thanh Kỳ - CH16PPGD Dương Việt Thông – GV ĐH KTQD


Bài T5/33: Giải hệ phương trình: Bài T14/33: Cho a , b là các số thực với a  b. Xét
 x 2  2  x  y  1 hàm f :[a , b ]  R liên tục, khả vi và có đạo hàm
 2 không đổi dấu trên ( a, b) . Chứng minh rằng tồn tại
 y  7  y  x  1 . 2 2
Trần Quốc Luật – 50A Toán c  (a , b ) sao cho  f (c)  .
a c bc
Bài T6/33: Cho các số thực x, y thỏa mãn điều Nguyễn Trần Thuận – CH17 XSTK
kiện x  y  0 . Tìm giá trị lớn nhất của biểu thức Bài T15/33: Cho f là hàm số khả vi liên tục trên
1
F  y( x 1) x 1  x( y 1) y 1  xy( x  y )  y  x.
Phạm Kim Chung – (cựu SV 41A1 Toán) 0;1 sao cho  f ( x)dx  0. Chứng minh rằng:
0
Bài T7/33: Cho các số thực x, y , z thỏa mãn điều 1
kiện xy  yz  zx  1 và  x  1 y  1 z  1  0. 1
2 2 2
 xf ( x)  12 max
0
  0;1
f '( x) .
x 1 y 1 z 1
Chứng minh: P     2. Dương Việt Thông – GV ĐH KTQD
x2  1 y2  1 z2  1
Bài T16/33: Cho A, B là 2 ma trận vuông cấp
Trần Quốc Luật – 50A Toán
Bài T8/33: Cho tam giác ABC. Chứng minh rằng: n  n  1 , các phần tử nhận giá trị trên trường K .
8cos 2 A cos2 B cos2 C  cos 2 A cos 2 B cos 2C  0. Ma trận A có n giá trị riêng phân biệt. Chứng
Trần Quốc Luật – 50A Toán minh AB  BA khi và chỉ khi tồn tại đa thức
P ( x)  K [ x ] sao cho B  P( A).
 
Bài T9/33: Cho   1 và các góc x, y , z , t   0;  Dương Xuân Giáp – GV khoa Toán, ĐHV
 2 Bài T17/33: Cho V là 1 không gian vector hữu hạn
sin x   sin y chiều trên trường số thực. Giả sử dimV  n. Tìm tất
thỏa mãn điều kiện: 
sin z   sin t. cả các tự đồng cấu f : V  V thoả mãn f  f  idV .
xz y t Nguyễn Anh Tuấn – SV ĐHKHTN-ĐHQGHN
Chứng minh rằng: sin   sin . Bài T18/33: Một cuộc thi bắn có 100 người dự
2 2
Đỗ Đức Hiếu – ĐH Ngoại Thương tuyển. Mỗi người được phát 5 viên đạn, họ bắn từng
Bài T10/33: Cho tam giác ABC nội tiếp đường tròn viên cho đến khi trúng mục tiêu 3 viên thì được coi
(O), ngoại tiếp đường tròn (I). Tiếp điểm của đường là qua vòng sơ tuyển. Giả sử xác suất bắn mỗi viên
tròn (I) với BC là D. Đường tròn đường kính AI cắt trúng mục tiêu của mỗi người dự tuyển là 0,5. Tìm
(O) tại M, cắt đường thẳng qua A song song với BC số k nhỏ nhất để sự kiện “Số người dự sơ tuyển đạt
tại N. Chứng minh rằng MO đi qua trung điểm DN. yêu cầu không vượt quá k người” có xác suất
Lê Phúc Lữ – ĐH FPT TP.HCM không nhỏ hơn 0,95.
Bài T11/33: Chứng minh rằng đa thức Nguyễn Thị Thanh Hiền – GV khoa Toán, ĐHV
Bài giải xin gửi về email: toanhocsinhvien@gmail.com hoặc nạp trực tiếp cho Trưởng BBT.
Thời hạn nhận bài giải: Từ ngày xuất bản đến hết 23h59’ ngày 26/03/2011.

TOÁN HỌC & SINH VIÊN số 33


19

GIẢI BÀI KỲ TRƯỚC


Bài T1/32: Cho 3 số thực a , b, c thỏa mãn quá 2 nghiệm. Vì f  0  f 1  0 nên phương
a  b  c  0 và abc  1. Chứng minh rằng: trình đã cho chỉ có 2 nghiệm là x  0 và x  1.
2 2 2
P  ab  a  b   bc  b  c   ca  c  a    18.1 Cách 2: (Phạm Thị Liên, Nguyễn Thị Nga) Giả sử
x0 là nghiệm. Xét hàm f (t )  t x0  tx0 liên tục trên
Cách 1: (Trần Quốc Luật) Đặt x  bc  b  c  ,
R có f (t )  x0 .t x0 1  x0 với mọi t. Theo Định lý
y  ca  c  a  , z  ab  a  b  . Ta có:
Rolle tồn tại c  (3;5) : f'(c)=0 hay x0 .c x0 1  x0  0 .
   xy  yz  xz    a  a  b  a  c   b  b  c  b  a 
Suy ra: x0  0; x0  1.
 c  c  a  c  b   a 3  b 3  c3  3abc  ab  a  b  Bài T3/32: Cho a , b, c  1 thỏa mãn
bc  b  c   ca  c  a   19a 2010  5b 2010  1890c 2010  2010.
3 Tìm giá trị nhỏ nhất của P  a  b  c.
 a3  b 3   a  b   6abc  9.
Lời giải: (Từ Hữu Sơn) Ta có:
2
Do vậy: P  x 2  y 2  z 2   x  y  z   18  18. 115 101
1  a  2010   , 1  b  2010  ,
Cách 2: (Đỗ Đức Hiếu) Bất đẳng thức (BĐT) cần 19 5
chứng minh tương đương với
1986
 ab
2
ab bc ca 15 1  c  2010  .
    18  2  2  2  . 1890
 ab c a b 4
Đặt m   2009   2008  ...   , ta dễ có
1 1 1
Đặt 3  x; 3  y; 3  z , ta có a 2010  m
a b c a 2010  m  (m  1)a  a  .
m 1
xyz  1; xy  yz  xz  a 3  b3  c 3  3abc  3. * Làm tương tự đối với  ,  rồi cộng các BĐT thu
15 1986
Khi đó, BĐT trở thành x  y  z   . được P  2  2010  2  x0 .
4 1890
Vì a  b  c  0 nên trong 3 số a,b,c có ít nhất 1 số
Vậy min P  2  x0 , khi a  b  1; c  x0 .
âm. Mà abc  1  0 nên trong 3 số trên có 2 số âm,
số còn lại dương. Không mất tính tổng quát giả sử Bài T4/32: Giải hệ phương trình
a  0  b; c thì x  0  y; z. Áp dụng BĐT Cauchy  x3  y 3  9 1
2  IV  :  2
ta có: ( y  z)2  4 yz  x( y  z)2  4  y  z   .
2
 x  2 y  x  4 y.  2
x Cách 1: (Đỗ Đức Hiếu) Ta có:
1  x 3  1 8  y 3
Kết hợp * suy ra x  . Do đó ta có:
4  IV   
 x( x  1)  2 y(2  y ).
2 1 15
x yzx  f ( x)  f     . Nếu x  1 thì y  2. Nếu x  1 thì y  2 . Khi đó
x 4
  4
x 2y
1 ta có: 2  2 , rút gọn suy ra
(do f '( x)  1   0, x ). x  x 1 y  2y  4
x x
Nhận xét: Giá trị nhỏ nhất của P là 18 khi  y  2 x  xy  2   0. Giải ra được
x  2, y  1.
a  3 4; b  c   3 2. Cách 2: (Phạm Thị Liên, Phan Minh Trí) Lấy cả 2
vế của phương trình  2  nhân với 3, sau đó lấy
Bài T2/33: Giải phương trình: 5 x  3x  2 x.
phương trình 1 trừ kết quả thu được, ta có:
Cách 1: (Phan Minh Trí, Đỗ Đức Hiếu) Đặt
f ( x)  5 x  3 x  2 x. Ta tính được x 3  y 3  3x 2  6 y 2  9  3 x  12 y hay
f '( x)  5 x ln 5  3x ln 3  2 và x 3  3 x 2  3 x  1   y 3  6 y 2  12 y  8
3 3
 
f ''( x)  2 5 x ln 5  3x ln 3 . Giả sử f ''( x) có tức là  x  1   2  y   x  3  y.
x Bạn đọc tự giải tiếp để tìm được đáp số.
5 0 3 Cách 3: (Trần Quốc Luật). Ta có
nghiệm x0 . Ta được    ln   , vô lý. Vậy
3 5
    
 x3  y 3  x  4 y   9 x 2  4 y 2 
f   x  vô nghiệm. Từ đó, theo Định lí Rolle, ta có  IV    2
f   x  có không quá 1 nghiệm và f  x  có không 
 
x2  4 y2  2
  x  4y .

TOÁN HỌC & SINH VIÊN số 33


20
Trừ vế theo vế 2 phương trình ta được Bài T7/32: Cho a , b, c là các số thực. Chứng minh
2 2
xy  2 x  y   2  2 x  y  . Giải tiếp như cách 1. a 2  bc
Nhận xét: Hệ đã cho tuy không đối xứng nhưng
rằng: S   2 2
cyclic 2a  2b  5c
2
 0.
cặp nghiệm của nó đối xứng (!).
a 2  bc
Lời giải: (Đỗ Đức Hiếu) Đặt s(a)  .
Bài T5/32: Cho ABCD là tứ diện gần đều. Gọi r 2a2  2b2  5c2
là bán kính mặt cầu nội tiếp tứ diện và R là bán 2a 2  b2  c 2
kính bé nhất trong các đường tròn ngoại tiếp các Ta có s( a )  . Viết 2 BĐT tương
mặt đáy. Chứng minh rằng: R  2 2 r . 
2 2a 2  2b2  5c 2 
Lời giải: (Phan Minh Trí, Đỗ Đức Hiếu) Dựng hình tự rồi cộng chúng theo từng vế ta có:
hộp chữ nhật ABDC . ABDC. Đặt AB  a; 4 3 2a 2  c 2
B D  b; AA  c. Ta có do ABCD là tứ diện gần 3 
S   
4  cyclic 2a 2  2b 2  5c 2
 P.

đều nên các mặt là các tam giác bằng nhau, do đó Áp dụng BĐT Bunyakovsky, ta có:
bán kính R cũng bằng nhau. Kí hiệu S ABD , Stp ,VX 16( a 2  b 2  c 2 ) 2
P  1.
lần lượt là diện tích của tam giác ABD , diện tích (2a 2  c 2 )(2 a 2  2b 2  5c 2 )

toàn phần của tứ diện ABCD và thể tích hình X . cyclic
Ta có: Vậy S  0. Đẳng thức xảy ra khi a  b  c.
1 Bài T8/32: Cho a , b, c là các số thực thỏa mãn
2
S ABD 
 a 2b 2  b 2 c 2  c 2 a 2 .
4
 ab  bc  ca  0. Chứng minh rằng:
AB.BD.DA a 2 (1  b)  b 2 (1  c)  c 2 (1  a )  abc(a  b  c  3)
Mà R  nên
4S ABD Lời giải: (Vũ Thanh Tú) Chú ý rằng:

R 2

a 2

 b 2 b2  c 2 c 2  a 2 . a 2b 2  b 2c 2  c 2 a 2  2abc(a  b  c). Ta có

 2 2
4 a b b c c a 2 2 2 2
 
2 a 2  b 2  c 2  a 2 b  b 2 c  c 2 a  abc( a  b  c  3) 
2 2 2
3VABCD 3VABCD  (a  b  bc)  (b  c  ca)  (c  a  ab)  0.
Ta lại có: r   và
Stp 4S ABD Nhận xét: Đẳng thức xảy ra khi (a, b, c)  (0,0,0)
1 hoặc ba nghiệm của phương trình t 3  3t 2  3  0.
VABCD  VAB DC . ABDC  4VABDB  abc.
3 Bài T9/32: Tính định thức sau:
2 2 2 2 x1  1 x2  1 ... xn  1
9V D a b c
Suy ra r 2  ABC  .

16S2ABD 4 a 2 b 2  b 2 c 2  c 2 a 2  x12 1 x 22  1 ... x n2  1
A x13 1 x 23  1 ... x n3  1
Áp dụng bất đẳng thức Cauchy ta có R 2  8r 2 hay
. . ... .
R  2 2 r (đpcm).
x1n  1 x2n  1 ... x nn  1
Bài T6/32: Giải phương trình
Lời giải: (Đinh Bích Yến) Ta có:
tan x sin x
 . 6 1 1 1 ... 1 2 1 1 ... 1
3  2 tan x 3  sin 2 x
Lời giải: (Trần Quốc Luật) Điều kiện: 1 x1 x2 ... xn 0 x1 x2 ... xn
cos x  2 tan x  3   0. Ta có 1 x12 x22 ... xn2 0 x12 x22 ... xn2
A 
 6   sin x 3  2sin 2 x   2sin x  3cos x    0. 1 x13 x23 ... xn3 0 x13 x23 ... xn3
2 2
. . . ... . . . . ... .
 sin x  cos x  3    sin x  2    0
  1 x1n x2n ... xnn 0 x1n x2n ... xnn
hay sin x  sin x  cos x  1  0. Đáp số: x  k .
Nhận xét: Với bản chất như trên, ta có thể sử dụng
  2 x1 x2 ...xn  ( x1  1)...( xn  1)    x  x .
n  i  j 1
i j

phương pháp thế, đánh giá, đặt ẩn phụ, bình phương Nhận xét: Bạn Đỗ Đức Hiếu sử dụng Định lý Bu-
chuyển về một biến,… Bạn Đỗ Đức Hiếu đánh giá zơ cũng cho lời giải đúng.
rất khéo léo: Ta thấy 3cos x  2sin x  2 nên cos x  0. Bài T10/32: Cho f , g là các hàm số dương và liên
Nếu sin x  0  3cos x  3  sin x(1  sin x)  3 (vô
tục trên [a, b] . Chứng minh rằng tồn tại c   a, b 
lý). Do vậy 3cos 2 x  2sin 2 x  3cos x  2sin x. f (c) g (c)
Giải tốt bài này còn có các bạn: Phan Minh Trí, sao cho   1.
c b
Phạm Thị Liên. 
a
f ( x ) dx 
c
g ( x ) dx

TOÁN HỌC & SINH VIÊN số 33


21
Giải: (Nguyễn Trần Thuận) Xét hàm Giải: (Dương Đức Lâm) Xét hàm số sau:
F ( x)  e x 
a
x
f (t )dt 
b

x
g (t )dt, ta có F ( a )  F (b)  0.   
g ( x)  f (1) x 2  f (0) x 2  1  f '(0) x x 2  1  f ( x). 
Áp dụng Định lý giá trị trung bình, ta có tồn tại Ta có g ( x) khả vi cấp 3 trên R và
c   a, b  sao cho F '(c )  0. Đây chính là đpcm.  
g '( x)  2 f (1) x  2 f (0) x  f '(0) 3 x 2  1  f '( x),
Bài T11/32: Cho hàm f :[  1;1]  R liên tục, có vi g ''( x)  2 f (1)  f (0)  6 f '(0) x  f ''( x),
phân cấp 2 liên tục trên [  1;1] . Giả sử có g '''( x)  6 f '(0)  f '''( x).
f (1)  f (0)  f (1)  0. Chứng minh c   1;1 Ta có g ( 1)  g (0)  g (1), nên theo Định lý Rolle,
để f (c ), f '(c) và f ''(c ) lập thành cấp số cộng. tồn tại c1   1;0  và c2   0;1 sao cho
Lời giải: (Đinh Bích Yến, Phan Minh Trí) Đặt g '(c1 )  g '(c2 )  0.
g ( x)  e  x f ( x). Khi đó g(x) liên tục trên [a, b]; khả Lại có g '(0)  0, nên tồn tại c3   c1;0  và
vi trên (a,b) và g ( 1)  g (0)  g (1)  0. Theo Định
c4   0; c2  sao cho g ''(c3 )  g ''(c4 )  0.
lí Rolle về giá trị trung bình ta có:
c1  ( 1,0) : g (c1 )  0; c2  (0,1) : g (c2 )  0. Theo Định lý Rolle, tồn tại c   c3 , c4  sao cho
Khi đó hàm g ( x)  e  x f ( x )  e x f ( x) cũng thỏa g '''(c)  0 hay f '''(c)  6 f '(0)  0. (đpcm)
mãn điều kiện của Định lí Rolle trên [c1 , c2 ] . Bài T14/32: Cho hàm f :  0;1  R liên tục trên
Ta có c  (c1 , c2 )  (1,1) để g (c)  0 0;1 , khả vi trên  0;1. Biết rằng f  0   f 1  0 .
e c f (c )  e c f (c ) f (c )  f (c) Chứng minh rằng tồn tại c   0;1 để
 f (c)   .
2e c 2 cf  c   1  c  f '  c  .
Vậy c  ( 1,1) để f (c ); f (c); f (c) lập thành cấp Lời giải: (Phan Minh Trí) Xét hàm
số cộng. g  x   e x 1  x  f  x  . Ta có g  x  cũng liên tục
Bài T12/32: Cho A là ma trận phản đối xứng cấp
trên  0;1 khả vi trên  0;1 . Theo Định lí Lagrange,
n, phần tử thực. CMR det I n  xA2  0 x  R.   g 1  g  0 
Lời giải: (Nguyễn Trần Thuận) tồn tại c   0;1 sao cho g '  c   0.
TH1: x  0 : Ta có 1 0
Mặt khác
  
det I n  xA2  det I n  i x A I n  i x A    g '  x   e x 1  x  f  x   e x f  x   e x 1  x  f '  x 
  
 det I n  i x A .det I n  i x A    e x 1  x  f '  x   xf  x  nên ta có

 det  I  i x A  .det  I i x A  cf  c   1  c  f '  c  (đpcm).


n n
Bài T15/32: Cho tam giác ABC có AB và AC có
 det  I  i
n x A  .det  I n i x A  phương trình lần lượt là x  4 y  11  0;
2
2x  y  4  0. Trung tuyến AM có phương trình
 det  I  i
n x A   0. x  5 y  7  0. Tính diện tích tam giác ABC biết
TH2: x  0 : Đặt y   x  0 . Ta có rằng đường thẳng BC đi qua điểm E  3;2  .
Lời giải: (Phan Minh Trí)Trên tia AM lấy I, J sao

det I n  xA 2
  det  I n  yA 2
 cho EI, EJ lần lượt song song với AC, AB. Ta có
  
 det I n  y A .det I n  y A .  IM

ME ME JM
  suy ra IM = JM. Do đó
AM MC MB AM
Vì A là ma trận phản đối xứng nên A   AT , suy ra M là trung điểm của IJ. Vì EI song song với AC nên
T 
I n  y A  I n  y AT  I n  y A . Do đó:   EI có vec-tơ pháp tuyến là n1   2; 1 . Vậy EI có
T  11 2 
 
det I n  xA2  det I n   y A  .det  I n  yA   phương trình là 2 x  y  8  0 . Suy ra I   ;  ,
 3 3
2
 
 det I n  y A   0.  1 4
J   ;  . Do đó M  2;1 . Suy ra BC : x  y  1  0.
 3 3
Bài T13/32: Cho hàm f :  1;1  R khả vi cấp 3
Tìm được B  5;4  , C 1; 2  , A  3;2  .
thỏa mãn f '(0)  0 và f (1)  f ( 1). Chứng minh  
rằng tồn tại c   1;1 để
f '''(c)
 6  0. Vậy S ABC 
det AB; AC  18.

f '(0) 2

TOÁN HỌC & SINH VIÊN số 33


22

TƯ DUY HÌNH HỌC TRONG ĐẠI SỐ


PHẠM THỊ LIÊN – 51A Toán
Tóm tắt. Bài viết Định hướng hình học trước một số bài toán đại số và sự tương giao giữa các phép biến
đổi hình học và phép biến đổi đại số.
Phương trình, hệ phương trình và bất đẳng thức là những dạng toán cơ bản trong các chuyên đề bồi dưỡng
học sinh trung học phổ thông. Bài viết này tôi sẽ giới thiệu đến các bạn một số bài tập được giải theo tư duy
hình học rất độc đáo. Thú vị hơn là từ đó ta lại có thể rút ra cách giải thuần tuý đại số rất mới mẻ cho một số
bài toán. Hãy cùng theo dõi những ví dụ sau:
I - PHƯƠNG TRÌNH, HỆ PHƯƠNG TRÌNH      
Nếu a  0 thì b và c cùng phương, suy ra c  2b .
Phương pháp: Đối với bài toán giải phương trình 1
Từ đó x  0; y  0; z   .
và hệ phương trình ta chủ yếu dùng công thức 2
khoảng cách và tích vô hướng của vectơ trong hình
Nhận xét: Cách giải bằng vectơ này khá hay và hiệu
học toạ độ. Xét một vài ví dụ minh hoạ sau:
quả. Tuy nhiên nhận thấy rằng từ mỗi bước bằng
phương pháp vectơ như trên ta có thể đưa ra lời giải
Ví dụ 1: Giải phương trình tương ứng mang thuần tuý đại số như sau: Biến đổi
2 2 hệ đã cho về hệ  I  .
x  4 x  5  x  10 x  30  5. 1
1
Giải: Ta có Nếu x  y  0 thì z  .
2
1  ( x  2) 2  1  ( x  5) 2  5  5. Nếu trái lại, giả sử y  0, ta có:

Trong mặt phẳng xOy, chọn  x( x  y)  y( y  z)  0


 .
 x ( x  1)  y ( 2 z  1)  0
A  2;1 , B  5;5  , M  x;0  thì 1 trở thành
MA  MB  AB. Mặt khác theo bất đẳng thức tam y z x y
Suy ra 
2z  1 x  1
giác thì MA  MB  AB. Đẳng thức xảy ra khi
5 4( x  y ) 2 4( x  y ) 2 4( x  y ) 2  4( y  z ) 2
M  AB  Ox. Đáp số: x  .   
4 ( x  1) 2 ( x  1) 2 ( x  1) 2  (2 z  1) 2
4( x  y )2  ( x  1) 2
Ví dụ 2: Giải hệ phương trình:  2 2
4( y  z )  (2 z  1) .
 x2  y 2   y( x  z)
 2 Đến đây bạn đọc dễ dàng giải tiếp.
 x  x  y  2 yz II – CHỨNG MINH BẤT ĐẲNG THỨC
 2 2
3 x  8 y  8 xy  8 yz  2 x  4 z  2. Phương pháp: Ngoài ứng dụng hai kiến thức trên
ta còn thường dùng đến phương trình, tính chất của
Giải: Hệ đã cho tương đương với:
đường tròn, elip, hypebol, tam giác… cùng các tính
 x( x  y )  y ( y  z )  0 chất liên quan.

 x( x  1)  y (2 z  1)  0 I  Ví dụ 3: Cho a , b, c  0. Chứng minh rằng:
 2 2 2 2
 4( x  y )  4( y  z )  ( x  1)  (2 z  1) . a 2  ab  b 2 + b 2  bc  c 2  a 2  ac  c 2 .
  
Xét a  x; y  ; b  x  y; y  z  ; c  x  1;2 z  1 . Khi đó Giải: Bất đẳng thức cần chứng minh được viết lại
 như sau:
 a.b  0
 
hệ (I) trở thành  a.c  0 b 3 c 3 2
 2 2 (a  ) 2  ( b )2 + (b  ) 2  ( c) 
 4b  c . 2 2 2 2

  1 a 2 3 2
Nếu a  0 thì x  y  0  z  .  (c  ) ( a) .
2 2 2

TOÁN HỌC & SINH VIÊN số 33


23
Đặt OA = a. OB = b, OC = c sao cho AOB  60 2 2
Giải: Nhận thấy  a  c    b  d  chính là khoảng
và BOC  60. Áp dụng Định lí hàm số cô-sin 4
trong tam giác ta có: cách giữa một điểm bất kì trên hypebol x  với
y
AB  a 2  ab  b 2 ; BC  b 2  bc  c 2 ; x2
một điểm trên elip  y 2  1. Vẽ hình tính toán ta
AC  a 2  ac  c 2 . Rõ ràng AB  BC  AC. 4
sẽ nhận ra điều phải chứng minh.
Ta có ngay điều phải chứng minh.
Nhận xét: Áp dụng BĐT Bunyakovsky ta có:
Ví dụ 4: Cho các số thực x, y, z thoả mãn:
 x  z 2   y  t 2  (1  4)   x  z  2 y  t 2
2 x  3 y  2  
 2
x  3y  9
 x  0, y  0.
 
và z 2  4t 2 1  1   z  2 t 

suy ra z  2 t  2 2.
Tìm GTLN của biểu thức P  x 2  y 2  4 x  8 y.
Áp dụng BĐT Cauchy ta
Giải: Bằng cách vẽ đồ thị ta thấy ngay tập hợp các
có: x  2 y  2 2 xy  4 2. Mặt khác
điểm M ( x; y ) với x, y thoả mãn bài ra là tứ giác
x  z  x  z và y  t  y  t . Do vậy
ABCD với A 1;0  , B  0; 2  , C  0,3 , D  9;0  .
2
2 2 2  x 2 y  z 2t  8
Khi đó xét Q  P  20   x  2    y  4   IM 2
 xz 2 yt  
5
 .
5
(với I  2;4  ) đạt GTLN khi M  D. Do vậy P đạt
Ví dụ 5: Cho các số thực x, y thoả mãn:
giá trị lớn nhất khi x  9; y  0. Suy ra max P  45.
Nhận xét: Từ tư duy hình học như trên ta có thể đưa  x 2  4 xy  6 y 2  2
 2
ra cách giải bài này hoàn toàn bằng đại số: Từ giả 2
6 y  8 yz  3 z  1.
0  x  9
thiết ta có điều kiện của x, y như sau  . Tìm giá trị lớn nhất của biểu thức P  xy  yz  zx.
0  y  3
2 2 Giải: Hệ đã cho được viết lại như sau:
Khi đó  x  2    9  2   49 đồng thời
2
2
 1  x2 1
 y  4  42  16 nên max P  45 khi x  9; y  0.  y  x   
 3  18 3
Ví dụ 3: Cho các số thực a, b thoả mãn  2
 2  z2 1
a 2  b 2  16  8a  6b. Chứng minh 7b  24 a.  y  z    .
 3  18 6
2 2
Giải: Ta có  a  4    b  3  32. Nhận thấy các Trong mặt phẳng, xét:
điểm M (a; b) thoả mãn bài toán là hình tròn tâm  x x   z 2x 
u y  ; ; v  ;y .
I (4;3) bán kính R  3. Xét đường thẳng  3 3 2  3 2 3 
24.4  3.7
d : 24 x  7 y  0 , ta có: d  I ;d   3R  1
24 2  7 2 Ta có u .v   xy  yz  zx  . Áp dụng bất đẳng
3 2
nên d tiếp xúc với O( I ;3). Vì Ox cũng là một tiếp   
thức u.v  u . v ta có ngay P  1.
tuyến và gốc toạ độ O  d nên hình tròn ( I ;3) nằm
hoàn toàn trong miền dương được chia bởi đường Nhận xét: Cách giải trên theo ngôn ngữ vec-tơ trong
thẳng d nghĩa là 24a  7b  0. hình học toạ độ, tuy nhiên thực chất nó được suy ra
Ví dụ 4: Cho a, b là các số thực thoả mãn từ bất đẳng thức Bunyakovsky nên ta có thể có lời
giải bằng đại số như sau:
ab  c 2  4 d 2  4. Chứng minh rằng:
Áp dụng bất đẳng thức Bunyakovsky ta có:
2 2 8 2
a  c  b  d   .  x z x  2x 
5 y   y  
 33 2 3 2  3 

TOÁN HỌC & SINH VIÊN số 33


24
 2 2 2 2 1. Cho các số thực a , b, c, d thoả mãn
x   x    z   2x  
  y       .     y   a 2  b 2  c 2  d 2  5. Tìm GTLN của biểu thức
  3   3 2    3 2   3  
  
1 P  5  a  2b  5  c  2 d  5  ac  bd .
 . Suy ra P  1.
18 2. Cho các số thực a , b. Chứng minh:

Như vậy với những ví dụ trên ta thấy trong nhiều a 2  4a  8 + 10b 2  18b  9
trường hợp khi phương pháp đại số gần như “bế
tắc” thì tư duy hình học lại tỏ ra hữu hiệu khi cho ta + a 2  2ab  10b 2  29.
những lời giải ngắn gọn mà không kém phần độc 3. Giải hệ phương trình:
đáo! Tuy nhiên, giữa đại số và hình học luôn có một
sự liên hệ mật thiết với nhau nên từ cách giải bằng ( x  2) 2  ( y  3) 2  ( y  3)( x  z  2)
hình học nhiều khi ta có thể đưa về thuần tuý đại số  2
 x  5 x  9 z  7 y  15  3 yz
và ngược lại. Dưới đây là một số bài tập để các bạn 8 x 2  18 y 2  18 xy  18 yz  84 x  72 y  24 z  176.
luyện tập: 

KHẢO SÁT TÍNH LIÊN TỤC ĐỀU CỦA HÀM MỘT BIẾN
TRẦN QUỐC LUẬT – 50A Toán

Tóm tắt. Bài viết khái quát 3 phương pháp khảo sát sự liên tục đều của một hàm số một biến thường dùng
trong phạm vi chương trình Toán A2, Khoa Toán, Đại học Vinh.

A – SỬ DỤNG ĐỊNH NGHĨA f ( xn )  f ( yn )  xn3  yn3 


Định nghĩa: Hàm f được gọi là liên tục đều trên
D  R nếu với mọi   0, tồn tại  ( )  0 mà 
xn2  xn yn  yn2 xn  yn  3.20102.  .
3.20102
với mọi dãy xn , yn  D thỏa mãn xn  yn   thì
Do vậy f ( x)  x3 liên tục đều trên D   0;2010  .
ta có f ( xn )  f ( yn )   .
Ví dụ 2: Chứng minh rằng hàm y  2 x  sin x  1
Thực sự đa số tân sinh viên đều ngại loại ngôn
ngữ ép-xi-lon này nhưng nếu để ý kỹ một chút thì liên lục đều trên R.
sẽ phát hiện ra bản chất đơn giản của vấn đề. Ta Nhận xét: Ta chỉ cần chứng minh
sẽ tìm cách chứng minh: y ( xn )  y ( yn )   xn  yn với mọi xn , yn  R.
f ( xn )  f ( yn )   xn  yn . Lúc này ta chỉ cần Thật vậy:
 y(xn )  y( yn )   2xn  sin xn 1   2 yn  sin yn 1
chọn   thì ta có điều phải chứng minh. Ta sẽ
  2  xn  yn   sin xn  sin yn 
làm rõ thông qua các ví dụ.
xn  yn x  yn
Ví dụ 1: Chứng minh hàm f ( x )  x3 liên tục đều  2  xn  yn   2 cos .sin n
2 2
trong khoảng D   0;2010  .
xn  yn x  yn
Phân tích: Ta có  2 xn  yn  2 cos sin n
2 2
f ( x n )  f ( y n )  x n3  y n3  x n2  x n y n  y n2 x n  y n .
xn  yn
Vì xn , yn  D   0;2010  nên 0  xn , yn  2010  2 xn  yn  2. sin
2
suy ra xn2  xn yn  yn2  3.20102. Suy ra
xn  yn
2  2 xn  yn  2.  3 xn  yn .
f ( xn )  f ( yn )  3.2010 xn  yn . 2
Vậy ta đã tìm ra   3.20102. Do vậy ta chọn 
Vậy   3 và ta chọn   .
  3
  . Lưu ý: Nếu Bất đẳng thức trên khó chứng minh
 3.2010 2
Lời giải: Với mọi dãy xn , yn  D thỏa mãn thì ta chuyển sang phương pháp tiếp theo đây.
B – SỬ DỤNG ĐỊNH LÝ CANTOR
 Định lý Cantor: Nếu hàm liên tục trên 1 đoạn thì
xn  yn  . Ta có:
3.20102 liên tục đều trên đoạn đó.

TOÁN HỌC & SINH VIÊN số 33


25
Ví dụ 3: Xét ví dụ 1 ở trên. Ta xét hàm không bị chặn thì ta chọn yn là nghiệm của nó,
0 ; x0 tức là f ( yn )  0.
 3
F ( x)   x ; 0  x  2010 Ví dụ 5: Xét tính liên tục đều của hàm f ( x)  ln x
 3
 2010 ; x  2010. trên khoảng D   0;1 .
Rõ ràng lim F ( x)  0  F (0) và Bước 1: Chọn 2 dãy  xn  ,  yn    0;1 . Ta có
x0
3 xn
lim F ( x)  2010  F (2010) nên F ( x) liên tục f ( xn )  f ( yn )  ln .
x 2010
yn
trên đoạn 0;2010. Theo Định lý Cantor hàm
xn
F ( x) liên tục đều trên đoạn 0;2010  , mà Bước 2: Giải hệ xn  yn  0 và ln  1 ta chọn
yn
D   0;2010    0; 2010 nên F ( x) liên tục đều e 1
trên D, hay f ( x) liên tục đều trên D. được nghiệm xn  ; yn  .
n n
Ví dụ 4: Xét tính liên tục đều của hàm e 1
Lời giải: Chọn dãy xn  ; yn  với
2
ex 1 n n
f ( x)  trên khoảng D   0;1 . n  3, 4,5... thỏa mãn  xn  ,  yn    0;1 .
x  x  1
Phân tích: Ta có Ta có: xn  yn  0 khi n  . Nhưng
x2 2 f ( xn )  f ( yn )  1  0. Do vậy, hàm f ( x) không
e 1 x 1
lim f ( x)  lim  lim  ;
x0 x 0 x  x  1 x  0 x  x  1 2 liên tục đều trên D   0;1 .
2
ex  1 e  1 Ví dụ 6: Xét tính liên tục đều của hàm
lim f ( x)  lim   c.
x 1 x 1 x  x  1 2 f ( x)  x sin x trên khoảng D   0;   .
1 ; x 0 Phân tích: Bài này biểu thức f ( xn )  f ( yn ) rất
 khó đánh giá nhưng nhận thấy hàm f ( x)  x sin x
Do vậy ta xét hàm F ( x)   f ( x) ; 0  x 1 .
c ; x 1 có nghiệm vô hạn không bị chặn. Do đó ta chọn

yn  2n , n  1, 2,3... Vì xn  yn  0 nên ta chọn
Rõ ràng hàm này liên tục nên lập luận tương tự 1
xn  2n  . Ta thử đánh giá:
như trên ta có f ( x) liên tục đều trên khoảng n
D   0;1 .  1  1
f ( xn )  f ( yn )   2n   .sin  2 n  
 n  n
C – CHỌN DÃY ĐỂ CHỨNG MINH HÀM
KHÔNG LIÊN TỤC ĐỀU  1 1  1 1
  2n   sin   2 n   .  2 .
 n  n  n  n
Điều kiện liên tục đều yêu cầu mọi dãy thỏa mãn Như vậy ta đã chọn được 2 dãy thỏa mãn điều
nên ở đây muốn chỉ ra không thỏa mãn ta chỉ cần
kiện * .
chỉ ra một dãy không thỏa mãn là kết luận luôn
được hàm không liên tục đều. Các phép biến đổi D – BÀI TẬP ÁP DỤNG
mang nặng tính lý thuyết nên nếu chọn được dãy Khảo sát tính liên tục đều của các hàm số sau:
coi như bài toán được giải quyết. Sau đây là thuật
1. f ( x)  x , x  1;   .
toán của tác giả bài viết này dùng để chọn dãy
chứng minh hàm không liên tục đều: 1
2. f ( x)  , x   0;1.
Bước 1: Chọn 2 dãy  xn  ,  yn   D và tính: x

f ( xn )  f ( yn ) . 3. f ( x)  sin , x   0;1.
x
Bước 2: Giải hệ phương trình:
1
 xn  yn  0 4. f ( x)  e x .cos , x   0;1.
  L  0  .  * x
 f ( xn )  f ( yn )  L. 5. f ( x)  arctan x, x  R.
c E – TÀI LIỆU THAM KHẢO
Thường thì chọn L  1; xn  yn   0 khi
n [1] Lê Viết Ngư, Toán cao cấp Tập 2, NXBGD,
n    c  const  và nếu f ( x) có họ nghiệm 1998.

TOÁN HỌC & SINH VIÊN số 33


26

VECTƠ  KHÔNG
TRÌNH HOÀI NAM – Cựu SV 44A1 Toán
Như chúng ta đã biết, vectơkhông trong mặt phẳng là vectơ cùng phương với mọi vectơ. Ngược lại,
nếu có một vectơ cùng phương hoặc cùng vuông góc với 2 vectơ khác phương thì đó phải là vectơ không.
Tính chất trên không có gì là xa lạ đối với học sinh. Lợi dụng điểm này ta có thể giải một số bài toán về
vectơ trong chương trình THPT.

 
1. Tính chất: Tương tự ta cũng có u.OB  0  Đpcm.

Tính chất 1: Nếu vectơ x cùng phương với 2 Bài toán 2: Cho ABC nhọn có trực tâm H. CMR:
 
vectơ không cùng phương thì x  0 .    
 tan A.HA  tan B.HB  tan C.HC  0 .
Tính chất 2: Cho 2 vectơ không cùng phương a Lời giải: Đặt
     A
  x.a  0   u  tan A.HA  tan B.HB  tan C.HC
và b . Nếu    thì x  0 . B1
 x.b  0 Ta có:
     
u.BC  tan C.CH .CB  tan B.BH .BC C1
2. Kiến thức bổ sung
      tan C.CA1.CB  tan B.BA1.BC. H
 
i) Tích vô hướng: a.b  a . b .cos a; b .
Để ý: tan B.BA1  tan C.CA1  AA1 . B C
 
ii) Công thức hình chiếu: Gọi C’, D’ lần lượt là Do đó u.BC  0 . A1
hình chiếu của C, D lên đường thẳng AB. Khi đó ta
    Hoàn toàn tương tự ta có
 
có: AB.CD  AB.C ' D ' . u. AB  0 . Từ đó suy ra đpcm.
3. Bài tập áp dụng Nhận xét: Thực ra bài toán trên nó đúng cho tam
Bài toán 1: Cho ngũ giác đều ABCDE tâm O. giác bất kỳ. Tuy nhiên để cho đơn giản, tác giả chỉ
Chứng minh rằng: nêu trong trường hợp tam giác nhọn. Trường hợp
      còn lại xin dành cho bạn
OA  OB  OC  OD  OE  0 . A
đọc chứng minh tương
     
Lời giải: Đặt u  OA  OB  OC  OD  OE tự.
Cách 1. Do ABCDE là ngũ giác đều nên B và E, C Bài toán 3: Cho ABC. E H B
và D đối xứng nhau qua OA. Do đó các tổng M là điểm bất kỳ nằm 3
    O
OB  OE , OC  OD là các vectơ có giá nằm trên trong tam giác. Gọi Sa,
  Sb, Sc lần lượt là diện 1 2
OA. Chính vì vậy u cùng phương với vectơ OA .
 tích của các tam giác
Hoàn toàn tương tự ta cũng có u cùng phương với MBC, MCA, MAB.
 D
C
vectơ OB . Từ đó suy ra đpcm. Chứng minh rằng M
   
Cách 2. Đặt OA = OB = OC = OD = OE = a. Sa .MA  Sb .MB  Sc .MC  0 A
Dễ có: Lời giải: Đặt
   
AOB  BOC  COD  DOE  EOA  720. u  Sa .MA  Sb .MB  Sc.MC Khi
 
Khi đó: đó: u.MA  S a .MA2 
   2             M3
u.OA  OA  OB.OA  OC .OA  OD.OA  OE.OA Sb MA.MB  Sc MA.MC. 1
2 C
 a 2  2 a 2 cos720  2a 2 cos1440.  S a .MA2  Sb .MA.MB.cos M1 B
   Sc .MA.MC.cos M 3 .

 u.OA  a 2  2a 2 cos720  cos1440 
1
Sử dụng công thức diện tích S  ab sin C , ta có:
Xét tổng S  cos 720  cos1440 , ta có: 2
 
2sin 300.S  2sin 360 cos720  2sin 360 cos1440  u.MA  MA2.MB.MC. sin M2  sin M3 cos M1  sin M1 cos M3 
 sin(300 )  sin1080  sin( 1080 )  sin1800 Trong đó: sin M3 cos M1  sin M1 cos M3  sin M1  M3  
1  
 S    u.OA  0 .
2  
sin 3600  M 2   sin M 2 .

TOÁN HỌC & SINH VIÊN số 33


27
      
Do đó: u.MA  0 . Chứng minh tương tự ta được Lời giải: Đặt u  OA  OB  OC  OH . Gọi M là
   
u.MB  0 . Do đó u  0 . trung điểm BC.
         
Bài toán 4: Cho ABC. Gọi AA1 , BB1 , CC1 lần lượt Ta có: u.AB  OA. AB  OB. AB  OC. AB  OH . AB
là các đường cao. Chứng minh rằng: Sử dụng công thức hình chiếu ta có:
         
Nếu AA1  BB1  CC1  0 thì ABC đều. u.AB  MA '. AB  MB.AB
             
Lời giải: Đặt u  AA1  BB1  CC1 . Do u  0 nên  
 MC. AB  MA '. AB  MB  MC AB  0.
    
u.BC  0 . Lại có: (do M là trung điểm BC nên MB  MC  0 ).
     
u.BC  BB1.BC  CC1.BC  BB12  CC12  BB1  CC1  
Tương tự u.BC  0 . Từ đó suy ra đpcm.
Tương tự ta sẽ suy ra:
2S 2S 2S 5. Lời kết:
AA1  BB1  CC1     a  b  c.
a b c Do khuôn khổ của bài báo có hạn nên tác giả chỉ
Bài toán 5: Cho ABC và một điểm O nằm trong trình bày trong mặt phẳng. Kết quả hoàn toàn có thể
mở rộng trong không gian. Xin dành cho bạn đọc
tam giác. Gọi A’, B’, C’ lần lượt là hình chiếu của O
nghiên cứu. Sau đây là một số bài tập dành cho bạn
lên BC, CA, AB. Đặt:
 BC   CA   AB  đọc:
a1  .OA ', a2  .OB ', a3  .OC ' . Bài toán 7. Cho tam giác đều ABC tâm O. M là
OA ' OB ' OC '
    điểm bất kỳ trong tam giác. Gọi D, E, F lần lượt là
Chứng minh rằng: a1  a2  a3  0 . hình chiếu của M lên BC, CA, AB. Chứng minh
       3 
Lời giải: Đặt u  a1  a2  a3 . rằng: MD  ME  MF  MO .
         2
  
Khi đó: u.AB  a1  a2 . AB  a1  a2 CB  CA   Bài toán 8. Cho  ABC có tâm đường tròn nội tiếp
   
          là I. Chứng minh rằng: sin AIA
. sin BIB .  0.
. sinCIC
  
 a2.CB  a1.CA  a2 .CB.cos a2;CB  a1 .CA.cos a1;CA 
  Bài toán 9. Cho ABC. Gọi A’, B’, C’ lần lượt là
Dễ thấy a1  CB, a2  CA và các tiếp điểm của đường tròn nội tiếp ABC với các
    cạnh BC. CA. AB. Chứng minh rằng, ABC đều nếu
  
cos a2 ; CB  cos a1; CA     
AA '  BB '  CC '  0 .
   
Suy ra u.AB  0. Tương tự u.BC  0  Đpcm. Bài toán 10. Cho ABC, M là điểm nằm trong tam
Nhận xét: Bài toán hoàn toàn có thể mở rộng ra cho giác. Gọi H, I, K lần lượt là hình chiếu của M lên
đa giác lồi. Và đó chính là Định lý “Con nhím”, các BC, CA, AB. Chứng minh rằng M là trọng tâm
    
vectơ ai được gọi là các “lông nhím”. HIK khi và chỉ khi a 2 MA  b 2 MB  c 2 MC  0 .

Bài toán 6: Cho ABC. Gọi H, O lần lượt là trực ĐẤU TRƯỜNG TOÁN HỌC SINH VIÊN
tâm và tâm đường tròn ngoại tiếp tam giác ABC.
   
Chứng minh rằng : OH  OA  OB  OC . Tác giả thách đấu: TS. Kiều Phương Chi.
Thời hạn thách đấu: đến hết ngày 26/03/2011.
A Bài toán thách đấu: Tồn tại hay không đơn ánh
f :[0; )  [0; ) thỏa mãn điều kiện:

 x2   y2 
f  f    k f ( x)  f ( y ) .
H 1 x   1 y 
với k  (0;1) cho trước.
O
Xuất xứ bài toán: Sáng tác.
B C
Địa chỉ nhận bài : toanhocsinhvien@gmail.com
A' M

TOÁN HỌC & SINH VIÊN số 33


28

LỜI THẦY NGÀY ẤY


TRẦN THỊ THANH NHÀN – 48A Toán

“Khi được biết em thích và muốn chọn nghề tôi vẫn là lời dặn của Thầy: “Nghề này cốt ở cái
sư phạm, Thầy rất vui và hạnh phúc, bởi trong tâm của mình, chữ tâm rất cần đối với một nhà
số thế hệ học trò bây giờ, vẫn có những người giáo chân chính. Em hãy cố gắng để trở thành
yêu nghề dạy học và có lý tưởng như em. một nhà giáo tốt.”
Nhưng bên cạnh niềm vui và hạnh phúc đó, Ôm ấp niềm vui xen lẫn những lo lắng, tôi
đấy là sự trăn trở của Thầy đối với những khó bắt đầu bước những bước đầu tiên trên con
khăn, vất vả sẽ đến với em một mai này. Bởi đường mình đã hằng mơ ước. Quả đúng như lời
Thầy biết, Thầy đã từng trải qua, và đã gắn bó của Thầy, nó không hề đơn giản như tôi nghĩ.
với nghề bấy nhiêu năm. Nhưng Thầy nghĩ Những vất vả ban đầu trong quá trình học đã
rằng, một khi đã yêu nghề thì bản thân mỗi khiến tôi dường như nản chí, đôi lúc nghĩ rằng,
người sẽ tự tìm thấy niềm vui trong công việc, phải chăng đã có sự nhầm lẫn nào đó trong sự
dù việc có vất vả, khó nhọc như thế nào chăng lựa chọn của mình. Nhưng rồi được sự dìu dắt
nữa. Thầy luôn tin và hy vọng rằng em sẽ và giảng dạy tận tình của các thầy cô, tôi đã
thành công trên bước đường của mình. Cố lên quen dần với môi trường học mới. Thầy cô
em!...” Khoa tôi nhiệt tình và rất quan tâm tới sinh
Đấy là lời dặn dò và động viên của người viên. Qua những bài giảng, chúng tôi được biết
Thầy dạy Toán tôi hồi trung học, cho đến giờ thêm nhiều điều, những bài học làm người và
nó vẫn vang lên trong tiềm thức, để rồi trở nhiều kiến thức mới để chuẩn bị hành trang
thành nguồn động lực chính thúc giục tôi cho mình sau này.Giờ đây, khi đã đi gần trọn
không ngừng cố gắng để thực hiện được ước hết con đường. 3 năm không phải là nhiều,
mơ của mình. nhưng cũng là vừa phải để mỗi chúng ta nhận
Hồi còn nhỏ, mỗi khi theo mẹ đến trường ra rằng, cái hành trang mang trên vai ấy, đang
(mẹ tôi trước đây là một giáo viên tiểu học), ngày một nặng hơn, nhiều thêm về kiến thức,
được nghe cô giảng những bài học đầu đời, về sự chính chắn trong suy nghĩ hay những đạo
trong tôi đã mơ ước một ngày nào đó, mình lý làm người…Sẽ khó có thể gọi là đủ, nhưng
cũng sẽ trở thành một cô giáo đứng trên bục nó cũng giúp chúng ta vững tin hơn, khi một
giảng, giảng bài cho học sinh. Mơ ước nhỏ bé mai này bước trên đường đời. Thầm cảm ơn
của thời bồng bột ấy ngày càng lớn dần, khi thầy cô, những người đang ngày đêm miệt mài
hàng đêm thấy mẹ miệt mài bên trang giáo án, với công việc, lặng lẽ âm thầm chở từng chuyến
ngày đến trường…Trong bao nhiêu bộn bề đò qua sông, và lặng nghĩ về Thầy, người đã
công việc ở trường, lớp, rồi về nhà, vậy mà mẹ cho tôi niềm tin để cố gắng, tôi tự nhủ rằng,
chưa một lần phàn nàn mệt mỏi. Mẹ thường mình cần phải phấn đấu hơn nữa.
nói: “Nghề giáo tuy hơi vất vả nhưng mẹ vẫn Vẫn biết giờ đây, những gì đang chờ đợi
cảm thấy hạnh phúc với nghề”. Có lẽ cũng mình phía trước còn nhiều, rất nhiều những
chính vì tâm huyết với nghề và tận tụy với học khó khăn. Cũng không dám chắc bản thân có
sinh như vậy, nên khi mẹ xa tất cả mọi người, đủ tự tin và khả năng để bước tiếp và thực hiện
hình ảnh mẹ vẫn ở lại mãi trong lòng những tốt những dự định. Nhưng ước mơ năm nào và
đồng nghiệp và bao lớp học trò. Ngày đó, Thầy lời dặn của Thầy luôn nhắc nhở tôi tiếp tục cố
chính là người đã giúp đỡ tôi rất nhiều trong gắng.
học tập và cuộc sống. Thầy đã động viên tôi Cũng đã bao lần tôi tự hỏi, mai này ra
học, ân cần chỉ bảo và nâng đỡ mỗi khi tôi vấp trường, khi đã vững tin đứng trên bục giảng,
ngã hay nản chí. Hình ảnh Thầy đã in sâu vào tôi có diễm phúc được học trò quý mến như tôi
tâm trí tôi từ ngày ấy. và bao trò khác đã quý mến Thầy hay không,
Lên cấp ba, tuy không được học với Thầy nhưng tôi vẫn luôn nghĩ đến lời Thầy: dù khó
nữa, nhưng mỗi khi Tết đến hay hè về, bạn bè khăn và vất vả thế nào thì đó đều không phải là
và tôi lại đến thăm Thầy, cùng ngồi nghe Thầy lý do làm vơi đi lòng yêu nghề cùng sự tâm
kể chuyện trong cuộc đời nhà giáo của mình. huyết, nhiệt tình của một nhà giáo. Và nếu
Nghĩ đến một ngày nào đó mình cũng sẽ được may mắn có được diễm phúc ấy, thì tôi cũng
như Thầy lại càng nuôi lớn ước mơ trong tôi. không ngần ngại mà nói với trò những lời
Ngày nhận được giấy báo của trường Đại giống như Thầy đã căn dặn tôi ngày trước.
học Vinh, tôi vui mừng báo tin cho Thầy. Hãy không ngừng cố gắng để thành công,
Trong niềm vui hân hoan ấy, thổn thức trong bạn nhé!

TOÁN HỌC & SINH VIÊN số 33


29

KINH NGHIỆM HỌC MÔN


TÔPÔ ĐẠI CƯƠNG
NGUYỄN DUY DIỆN – 49A Toán

Tôpô đại cương là môn toán cơ sở quan trọng trong chương trình toán đại cương, là cơ sở của giải tích
hiện đại. Đây cũng là học phần tiên quyết để học các học phần như Độ đo và tích phân, Giải tích hàm, Giải
tích nâng cao,… Do vậy, học tốt Tôpô là điều kiện cần để có thể học tốt các học phần khác liên quan.
Trong bài viết này tôi xin chia sẻ với các bạn một số kinh nghiệm trong học Tôpô đại cương, hy vọng
giúp ích phần nào cho các bạn.

1. Tài liệu học Tôpô


Ngoài giáo trình Tôpô đại cương của tác giả Nguyễn Xuân Liêm, có một số tài liệu học Tôpô không nên
bỏ qua:
[1] Tôpô đại cương, J.Kelly, 1976.
[2] Tôpô đại cương, Đậu Thế Cấp, NXB Giáo dục.
[3] Hàm số biến số thực, Nguyễn Định, Nguyễn Hoàng, NXB Giáo dục.
[4] Bài tập Tôpô đại cương, Nguyễn Nhụy, Lê Xuân Sơn, NXB Giáo dục.
Cuốn [1] có 1 quyển ở Kho Tự chọn, tầng 3, Thư viện. Cuốn [2] chỉ có thể tìm mua ở các quán Photo
xung quanh Đại học Vinh. Cuốn [3] và cuốn [4] có ở Kho Giáo trình, tầng 1, thư viện (mượn miễn phí).

2. Nắm vững lý thuyết


a. Hoạt động ngôn ngữ
Diễn đạt theo cách bạn thích, bằng chính ngôn ngữ của bạn không những chứng tỏ bạn hiểu mà còn giúp
khắc sâu nó vào trí nhớ. Phát biểu lại khái niệm, Định lý bằng lời lẽ của mình và biết phát biểu, diễn đạt
Định nghĩa, khái niệm, Định lý dưới nhiều dạng khác nhau. Các bạn có thể tham khảo ở các tài liệu tôi đề
cập ở trên.
Ví dụ: Khái niệm “điểm giới hạn”: Cho tập   A  X , x  X .
Khi đó x được gọi là “điểm giới hạn” (hay điểm tụ) nếu:
-  U x là lân cận của x thì U x \  x   A   
- Mọi lân cận của x đều chứa điểm khác x của tập A.
- Mọi “lân cận thủng” của nó đều cắt A.
b. Hình tượng hóa khái niệm, Định lý
Bản thân toán học là một sự thống nhất giữa cái cụ thể và cái trừu tượng. Tôpô cũng không ngoại lệ,
dường như những khái niệm rất trừu tượng của nó rất khó nắm bắt và ghi nhớ nếu như ta không minh họa
được dưới những hình ảnh cụ thể. Do vậy, việc hình tượng hóa khái niệm một cách sinh động và vui nhộn
sẽ giúp ta hiểu sâu và nhớ lâu hơn. Chẳng hạn khái niệm không gian Hausdorff ( T 2_không gian):

X được gọi là không gian chính quy nếu X là


không gian Tôpô có nhiều hơn 1 phần tử, và
x  X , F  X , F đóng mà x  F thì tồn tại
các lân cận (mở) Ux của x và VF chứa F sao
cho U x  VF  

Không gian Hausdorff là một không gian Tôpô


có nhiều hơn một phần tử, sao cho với mọi cặp
điểm phân biệt bất kỳ thuộc nó thì luôn tồn tại
lân cận mở của chúng không cắt nhau.

c. Hệ thống hóa các khái niệm, Định lý


Việc hệ thống hóa kiến thức giúp ta nhìn một cách tổng thể, hiểu mối liên hệ giữa các khái niệm, Định lý
khác nhau trong một tổng thể thống nhất.
Có một công cụ rất hiệu quả đó là Bản đồ tư duy (Mindmap). Theo đó, tư duy được thể hiện ra như một
sơ đồ, bắt đầu bằng hình ảnh trung tâm chủ đề và dần dần triển khai ý tưởng ra các nhánh. Bản đồ tư duy là
TOÁN HỌC & SINH VIÊN số 33
30
một công cụ đa năng, giúp phát huy tối đa sức mạnh của não bộ. Bạn có thể nghĩ, với môn Toán đại cương
thì lập bản đồ tư duy như thế nào? Toán là môn học logic, việc lập bản đồ tư duy có thể bắt đầu với những
khái niệm, chủ đề, nguyên lý, công thức… Trong phạm vi bài viết này tôi không thể đề cập hết cả cách lập
và sử dụng MindMapping, các bạn có thể tìm hiểu thêm trên Internet và các cuốn sách liên quan.
d. Luyện tập, luyện tập và luyện tập
“Con đường duy nhất để học toán là làm toán” (George Polya) Nắm vững lý thuyết chưa đủ để đảm bảo
bạn có thể đạt được thành tích cao. Một yếu tố khác cũng quan trọng không kém đó là khả năng ứng dụng lý
thuyết để giải các bài toán, câu hỏi, trong đó bạn phải phát huy một loạt kỹ năng suy nghĩ bao gồm phân
tích, lập luận, sáng tạo,… Điều này đúng cho bất kỳ môn học nào. Bạn có thể thu thập tất cả các dạng câu
hỏi, bài tập liên quan trong các đề thi năm trước, giáo trình và các sách tham khảo và luyện tập giải quyết
chúng cho đến khi thành thạo.
Một diễn giả từng nói: “Bí quyết thành công nằm trong 3 từ:
Rèn luyện
Rèn luyện
Và rèn luyện.”

**********************************************************************

MỐI LIÊN HỆ GIỮA HÌNH HỌC XẠ ẢNH


VÀ HÌNH HỌC A-FIN
LÊ NHƯ HẢO – 49A Toán

Hình học xạ ảnh là một ngành toán học có ứng ánh xạ). Các đối tượng khác cũng có điểm tương
dụng quan trọng trong hội họa và kiến trúc. đồng, từ hệ điểm độc lập, mục tiêu, phẳng. Mục tiêu
Đặt nền móng cho sự ra đời của môn hình học Afin và mục tiêu xạ ảnh nhìn về tổng thể thì chỉ
này là hai nhà toán học người Pháp Gerad khác nhau ở chỗ gốc mục tiêu trong Afin được thay
Desargues (1591-1661) và Blaise Pascal (1623- thế bằng điểm đơn vị trong xạ ảnh. Và ta cũng thấy
1662). Nhưng những nét sơ khai này của hai ông trong không gian xạ ảnh ta có thể xây dựng được
chưa được giới toán học ủng hộ cho tới hơn 1,5 thế mô hình Afin, và cũng có thể xây dựng được mô
kỉ sau, đánh dấu sự ra đời chính thức của hình học hình xạ ảnh của không gian Afin.
xạ ảnh bằng bộ sách “Bàn về tính chất xạ ảnh của Nói như thế để chúng ta thấy được mối liên hệ
hình vẽ” năm 1822, là công trình của nhà toán học mật thiết giữa hình học xạ ảnh và hình học Afin.
Pháp J.V.Poncelet (1788-1867) Vậy, khi học về hình học xạ ảnh, các bạn nên
Từ năm thứ hai, chúng ta đã được học môn hình vận dụng những kiến thức đã học từ Afin, vận
học Afin và hình học Ơclit; đó chính là tiền đề, là dụng, đối chiếu với hình học xạ ảnh, chúng ta sẽ
kiến thức nền để học tốt môn hình học xạ ảnh. Hình tiếp thu bài tốt hơn.
học xạ ảnh chính là môn học nối tiếp của môn hình Và về cách học không chỉ xạ ảnh, Afin, mà tất cả
học Afin và hình học Ơclit. các môn khác, mình phải có sự đối chiếu, so sánh
Theo cách nhìn của tôi, hình học xạ ảnh và hình với các môn đã học để tiếp thu bài tốt hơn, và
học Afin có mối quan hệ tương quan với nhau, khi những kiên thức nền đã quên thì khẩn trương phải
thay đổi một số thành phần thì tính chất cũng như học lại. Bạn cũng nên cụ thể hóa các kiến thức ta
một số đối tượng trong Afin sẽ như của xạ ảnh. học bằng các ví dụ cụ thể, gần gũi hơn, ví dụ như
Xạ ảnh là một môn học khó, nó trừu tượng như quy về các không gian một, hai, ba chiều để hiểu
Tôpô, chứ không còn cụ thể như hình học Afin và hơn vấn đề.
hình học Ơclit. Thế nhưng biết liên hệ với những Các bạn cũng cần phải làm nhiều bài tập. Có thể
kiến thức đã học từ hình học Afin, thì chúng ta thấy tham khảo các cuốn bài tập:
hình học xạ ảnh không quá xa lạ. [1] Văn Như Cương , Bài tập hình học Afin và
Hình học Afin thì đối tượng nghiên cứu xoay hình học Ơclit.
quanh điểm, còn xạ ảnh là véctơ đại diện cho điểm, [2] Nguyễn Hữu Quang, Trương Đức Hinh,
nó cũng có chút khác nữa là trong hình học xạ ảnh Bài tập hình học xạ ảnh.
không có quan hệ song song. [3] Phạm Khắc Ban, Phạm Bình Đô, Hình học
Thế nhưng giữa chúng có rất nhiều điểm chung. Afin và hình học Ơclit qua các ví dụ và bài tập.
Từ Định nghĩa, hai không gian đều được tạo thành [4] Nguyễn Mộng Hy, Hình học cao cấp.
từ ba đối tượng (tập điểm, không gian véctơ, một Chúc các bạn thành công!

TOÁN HỌC & SINH VIÊN số 33


31

HƯỚNG DẪN SỬ DỤNG LATEX


TRẦN QUỐC LUẬT – 50A Toán

Có thể nói MikTeX là hệ thống TeX tốt nhất hiện nay, hơn thế nữa đây là phần mềm mã nguồn mở, hoàn
toàn miễn phí. Để cài đặt và sử dụng MikTeX, đầu tiên vào đây http://miktex.org/2.9/setup để download file
setup về và cài đặt theo mặc Định vào C:\Program Files\MiKTeX 2.9 trong menu Start sẽ có, bạn chọn
Setting. Trong hộp thoại bạn chọn như hình và nhấn nút Refresh FNDB, Update Formats, rồi OK (nhớ chờ
máy tính xử lý cho từng lệnh).

Cho đến lúc cài đặt xong, yêu cầu khởi động lại máy. Để sử dụng ta click chuột như sau:

Bước 1: Chọn Start\Programs\MikTeX 2.9>TeXworks. Hiện cửa sổ TeXworks.


Bước 2: Copy đoạn code sau dán vào khung soạn thảo của TeXworks:

\documentclass[12pt]{article}
\usepackage[utf8]{vietnam}
\usepackage{amsmath,amsxtra,amssymb,latexsym,amscd,amsthm}
\usepackage[a4paper,left=1.5cm,right=1.5cm,top=1cm,bottom=1.5cm]{geometry}
\renewcommand{\baselinestretch}{1.2}
\begin{document}
BẤT ĐẲNG THỨC CÔ SI (CAUCHY)\\
\[\dfrac{x_1+x_2+......x_n}{n}\ge \sqrt[n]{x_1.x_2...........x_n}\]
Dấu “ $=$ ” xảy ra khi và chỉ khi: $x_1=x_2=............=x_n$
\end{document}

Bước 3: Bấm Ctrl+S để lưu lại file .tex. (chú ý folder đã lưu file tex).
Bước 4: Chọn bên khung đổ xuống "pdflatex" rồi click nút Typeset (màu xanh lá).

TOÁN HỌC & SINH VIÊN số 33


32

Giải thích các dòng lệnh:


1. Khai báo lớp tài liệu dạng bài báo cỡ chữ 12pt.
2. Khai báo gói tiếng Việt mã utf-8 (Unicode).
3. Khai báo các gói dùng cho ký hiệu toán học.
4. Khai báo gói xác định cỡ giấy A4, lề trái 1.5cm, lề phải 1.5cm, lề trên 1cm, lề dưới 1.5cm.
5. Đặt lại lệnh dãn dòng 1.2 (mặc Định là 1).
6. Bắt đầu tài liệu.
7. Gõ tài liệu bằng tiếng Việt với mã unicode dựng sẵn. Dấu \\ để chỉ xuống dòng.
8. Dấu \[ \] để đưa công thức toán ở giữa trang.
9. Dấu $ $ để chèn công thức toán trong dòng chữ.
10. Kết thúc tài liệu.

Sau đây là các ví dụ cơ bản về công thức Toán:

Công thức Code Hiển thị


Phân số $\dfrac{x+2010}{x+2011}$ x  2010
x  2011
$a^n_i$ ain , a1n  a2n 1  a3n 2  ...  an2n1
Chỉ số $a_{1}^n+a_2^{n+1}+a_{3}^{n+2}+...+a_{n+1}^
{2n}$
$\sqrt{a}$ a , n x2  1 ,
Căn, mũ $\sqrt[n]{x^2+1}$
2010
$\sqrt[2010]{a^{2011}}$ a 2011 .
$\sum_{i=1}^k a_i^n$ k
n
n
k
Tổng, tích $\prod_{i=1}^{n} a_u^k$ a , a
i 1
i
i 1
u

Góc $\widehat{ABC}$ 
ABC
$\begin{cases}x-y=2\\ x+y=4\end{cases}$ x  y  2
Hệ PT 
x  y  4
$\left[\begin{array}x-y=2\\ x  y  2
Tuyển PT x+y=4\end{array}\right.$ x  y  4

$\left[ 3({{x}^{2}}+{{z}^{2}})+2xz \right]\left[  2 2
 2 2 
3({{y}^{2}}+{{t}^{2}})-2yt \right]=\left[ 3( x  z )  2 xz  3( y  t )  2 yt  
2{{(x+z)}^{2}}+{{(x-z)}^{2}} \right]\left[   2( x  z ) 2  ( x  z ) 2  ( y  t ) 2  2( y  t ) 2  
Biểu thức {{(y+t)}^{2}}+2{{(y-t)}^{2}} \right]\text{ }\ge 2
{{\left[ \sqrt{2}(x+z)(y+t)+\sqrt{2}(x-z)(y-t)   2 ( x  z )( y  t )  2( x  z )( y  t )  
 
\right]}^{2}}=8{{(xy+zt)}^{2}}=8.$
 8( xy  zt ) 2  8.

Các bạn muốn tìm hiểu sâu hơn có thể lên trang web http://google.com.vn để tìm giáo trình nhé!

TOÁN HỌC & SINH VIÊN số 33


33

HƯỚNG DẪN TẠO VIDEO CLIP ALBUM


NGUYỄN HUY HÙNG – 49A Toán
Bước 1: Bạn download phần mềm “QSYJ2008” và các Template của nó về. Bạn có thể download
tại: http://hungcoi.forumvi.com/forum-f9/ bao gồm cả chương trình cài đặt và các template có sẳn.
Bước 2: Cài đặt chương trình.
Bước 3: Bắt đầu làm clip.
Bạn nhấp chuột chạy chương trình QSYJ2008 sẽ xuất hiện hộp thoại sau. Bạn nhấp chuột chọn OK.

Xuất hiện hộp thoại sau:

Chọn để bắt đầu chọn ảnh

Chọn thư
mục chứa
ảnh

Chọn
Open

Click vào đây

TOÁN HỌC & SINH VIÊN số 33


34

Click vào đây


Để chọn Template

Chú ý: Template là các hiệu ứng đã dựng sẵn của chương trình này, file có dạng *.BWS. Bạn có
thể tự tạo hoặc tìm kiếm trên mạng tải về thêm:

Chọn tới
đường dẫn
có chứa các
Template

Click vào đây để


hoàn thành

Click vào đây


chọn thư mục
lưu

Xuất hiện hộp thoại: Chọn nơi lưu sau đó chọn OK. Vậy là bạn đã xong. Chương trình này xuất
ra file flash video dạng *.SWF. Bạn dùng chương trình đổi đuôi sang dạng video thường là được,
“dùng Swf2Avi” hoặc dùng Uleadstudio bạn có thể thay nhạc hoặc kết hợp với nhiều hiệu ứng của
các phần mềm khác.

TOÁN HỌC & SINH VIÊN số 33


35

CHUYÊN ĐỀ TÍCH PHÂN


ÔN THI OLYMPIC TOÁN SINH VIÊN
VÕ VIẾT CHƯƠNG – 50A Toán

Trong công tác chuẩn bị cho đợt thi toán sinh viên toàn quốc sắp tới, nhằm tổng hợp cho các bạn đam mê
về toán học và các bạn sinh viên khóa 51 mới nhập học khỏi bở ngỡ, tạp chí Toán học sinh viên xin đăng bài
viết về tích phân, một phần quan trọng của bài thi.
Phần tích phân này rất rộng, để có thể làm tốt phần này các bạn cần nắm vững các kiến thức cơ bản về
tích phân trong sách Toán A2 và A3 mà các bạn được học. Ngoài ra các bạn cần chú ý thêm về:
- Định lý giá trị trung bình của tích phân.
- Định lý không âm của hàm dưới dấu tích phân.
- Định lý về tính liên tục và khả vi của hàm dưới dấu tích phân; quy tắc tính giới hạn cũng như đạo
hàm của tích phân phụ thuộc tham số.
- Định lý Rolle, Lagrange.
- Bất đẳng thức Cauchy-Schwartz.
Từ dễ đến khó, từ khó đến dễ tùy vào cách học cũng như cách nhìn của các bạn, BBT xin đưa ra một số ví
dụ và cách giải để các bạn có cách nhìn mới. Để bài viết thực sự mang lại hiệu quả, chúng tôi xin các bạn
dành một ít thời gian để xem lại các Định lý đã nêu trên.
1. Tính liên tục, đơn điệu của hàm dưới dấu tích phân và Định lý giá trị trung bình của tích phân.
Ví dụ 1: Cho f là một hàm liên tục trên [0, b] và 0  a  b , f nghịch biến trên [0, b] . Chứng minh rằng:
a b

 
b f ( x)dx  a f ( x) dx
0 0
a a b a b
Lời giải: Ta có b f ( x)dx  a f ( x) dx  a f ( x)dx   b  a  f ( x) dx  a f ( x) dx.
    
0 0 a 0 a
Theo Định lý giá trị trung bình của tích phân, tồn tại 1 , 2 : 0  1  a   2  b thỏa mãn đồng thời
a b

 b  a   f ( x)dx  a  b  a  f (1 ) và a f ( x)dx  a  b  a  f ( 2 ).



0 a
a b
Vì f (1 )  f ( 2 ) nên b f ( x) dx  a f ( x) dx.
 
0 0
Ví dụ 2: Cho f, g là các hàm liên tục đơn điệu cùng loại trên [a,b]. Chứng minh rằng:
b b b b
1

a
f ( x) g (a  b  x) dx 
ba a  
f ( x)dx g ( x) dx  f ( x) g ( x) dx
a a

Hướng dẫn: Theo Định lý giá trị trung bình của tích phân, tồn tại x0 [a, b ] thỏa mãn
b b


a

g ( x) dx  g (a  b  x) dx  g (a  b  x0 ) .
a

Sử dụng  f ( x)  f ( x0 )   g  a  b  x   g (a  b  x0 )   0, x [a, b] để suy ra đpcm.


2. Bất đẳng thức Cauchy-Schwartz
Cho f, g là các hàm liên tục trên [a,b]. Chứng minh rằng:
2
b  b b
2
 f ( x) g ( x )dx   ( f ( x)) 2 dx.  g  x   dx
  
 
a  a a
b
2
Hướng dẫn: Từ   t. f ( x)  g ( x ) 
a
dx  0, x [a, b] suy ra   0 ta có đpcm.

Ví dụ 3: Cho f là hàm khả vi liên tục đến cấp 2 trên [0,2]. Chứng minh rằng:
2
2 3
 f 
"
( x) dx  ( f (0)  2 f (1)  f (2)) 2 .
0
2

TOÁN HỌC & SINH VIÊN số 33


36
Lời giải: Ta có:
2
1 1 1
2 21  2
 f     
" 2
( x) dx  3 x dx  f ( x) dx  3  xf " ( x) dx   3 f ' (1)  f (0)  f (1) ,
"

 
0 0 0 0 
2
2 2 2
2 2  2
 f    x  2  dx     
" 2
( x) dx  3 f ( x) dx  3   x  2  f " ( x )dx   3  f ' (1)  f (2)  f (1) .
"

 
1 1 1 1 
Do đó
2
2 2 2 3
 f ( x) dx  3  f ' (1)  f (0)  f (1)   f ' (1)  f (2)  f (1)    f (0)  2 f (1)  f (2)  .
    
" 2
  2
0

Ví dụ 4: Cho a  [0,1] . Tìm tất cả các hàm không âm, liên tục trên [0,1] sao cho:
1 1 1
2
 f ( x) dx  1,  xf ( x) dx  a, x f ( x) dx  a 2 .
0 0 0
Hướng dẫn: Sử dụng bất đẳng thức Cauchy-Schwartz:
1 1 1


a  xf ( x) dx   x 2 f ( x )dx .  f ( x)dx  a, dấu “=” không xảy ra nên không có hàm f nào thỏa mãn.
0 0 0
3. Tính không âm của hàm dưới dấu tích phân
b
Ví dụ 5: Cho f là hàm liên tục trên [0,1] thỏa mãn  f ( x ) dx  0. Chứng minh rằng f ( x)  0 với mọi
a
x [a,b] .
Lời giải: Giả sử tồn tại x0 [a, b] sao cho f ( x0 )  0 . Ta có f ( x0 )  0. Do tính liên tục của f, tồn tại
b 
 ,     a, b và   0 sao cho f ( x)   , x   ,   . Khi đó  f ( x) dx   f ( x)dx   (   )  0.
a 

Mâu thuẫn này chứng tỏ f ( x)  0, x  [a, b] .


b
Ví dụ 6: Cho hàm số f(x) xác định và liên tục trên đoan [a,b], (a<b) và thỏa mãn điều kiện f ( x) dx  0. 
a
c
Chứng minh rằng tồn tại c   a, b  sao cho f (c)  2010 f ( x)dx. 
a
t
Lời giải: Xét hàm số F (t )  e 2010t f ( x )dx. Khi đó F(a)=F(b)=0 và 
a
t
F ' (t )  2010e 2010t f ( x) dx  e2010 t f (t ).

a
'
Theo Định lý Rolle, tồn tại c   a, b  sao cho F (c)  0 nghĩa là:
c c
2010e 2010c f ( x) dx  e 2010c f (c)  0. Từ đây suy ra f (c)  2010 f ( x )dx . (đpcm)
 
a a

GIẢI TRÍ TOÁN HỌC: XEM AI NHANH HƠN!


Hãy giải những bài toán sau đây mỗi câu trong 1 phút!
1. Năm ngoái trong dàn đồng ca của trường số học sinh nam nhiều hơn số học sinh nữ 30 người. Năm nay,
thành phần đội đồng ca tăng lên 10%: số học sinh nữ tăng 20% và số học sinh nam tăng 5%.. Hỏi có bao
nhiêu học sinh trong dàn đồng ca của năm nay?
2. Tú xoá đi 1 trong 10 số nguyên dương liên tiếp. Tổng của 9 số còn lại bằng 2010. Hỏi Tú đã xoá số nào?
3. Làm thế nào có thể biết được trọng lượng của một chiếc máy bay mà không sử dụng bàn cân?
4. Tại sao hầu hết các nắp cống trên đường có hình tròn mà không phải là hình vuông?

TOÁN HỌC & SINH VIÊN số 33


Đội tuyển Olympic Toán Sinh viên toàn quốc trường Đại học Vinh tại thành phố Huế 04/2010
Ban chấp hành Liên chi Đoàn khoa Toán, trường Đại học Vinh nhiệm kỳ 2010-2012

Đoàn sinh viên trường Đại học Vinh tham gia Trường hè Toán học Viên Toán, hè 2010

You might also like